Qui si risolve LOGO
a

Menu

M

Chiudi

Funzioni convesse – Teoria

Funzioni Convesse

Home » Funzioni convesse – Teoria

Le funzioni convesse sono estremamente importanti nell’Analisi Matematica. Una funzione reale di variabile reale si dice convessa se le direzioni delle rette secanti al grafico sono monotone, e dunque variano “sempre nello stesso senso”. Tale proprietà possiede numerosi risvolti di carattere sia teorico che pratico, utili in numerose applicazioni. Questa dispensa vuole offrire uno studio della convessità e delle sue conseguenze, concentrandosi sui seguenti punti fondamentali:

  • Come si definisce formalmente la convessità e quali sono le sue interpretazioni geometriche?
  • Come si caratterizza la convessità e quali sono le sue relazioni con la continuità? mostreremo infatti che le funzioni convesse sono continue all’interno dell’intervallo di definizione;
  • Come si caratterizza la convessità in relazione alla monotonia delle derivate prime e al segno delle derivate seconde?
  • Quali tipo di punti di estremo può avere una funzione convessa? Vedremo che, all’interno del suo dominio, essa può avere soltanto punti di minimo;
  • Cos’è un punto di flesso e quali sono le sue proprietà?
  • Applicazioni e approfondimenti: metodo di Newton per la determinazione degli zeri di un’equazione e disuguaglianza di Jensen, una generalizzazione della definizione di convessità.

Se desideri approfondire questi argomenti con una lettura dettagliata, semplice e chiara, questa dispensa è quello che cercavi!

Segnaliamo che alla fine della dispensa vi sono numerosi esercizi su questo importante argomento; riportiamo inoltre le seguenti dispense sulla teoria collegata:

 

Sommario

Leggi...

Questa dispensa è un’introduzione alla teoria di funzioni convesse reali di variabile reale. Partendo dalla definizione, vengono studiate le caratterizzazioni della convessità e il suo legame con continuità e derivabilità. Vengono poi trattati gli estremi di una funzione convessa, il concetto di punto di flesso, il metodo di Newton per funzioni convesse e la disuguaglianza di Jensen. Sono infine riportati alcuni esercizi svolti di carattere teorico e pratico.

 
 

Autori e revisori


 
 

Prerequisiti

Leggi...

Questa dispensa necessita della conoscenza della logica elementare (implicazione, equivalenza), della definizione intuitiva di insieme, le operazioni tra insiemi (unione, intersezione, prodotto cartesiano), e infine la definizione e le proprietà degli insiemi numerici più comuni. Inoltre, è necessaria la conoscenza delle funzioni elementari più comuni [2], come potenze, radici, esponenziali, logaritmi, seno e coseno, e dei concetti di continuità, derivabilità, massimi e minimi di una funzione [3, 5].

 
 

Notazioni

Leggi...

\emptyset insieme vuoto;
\mathbb{N} insieme dei numeri naturali;
\mathbb{Z} insieme dei numeri relativi;
\mathbb{Q} insieme dei numeri razionali;
\mathbb{R} insieme dei numeri reali;
f'_{-}(x_0), f'_{+}(x_0) rispettivamente derivata sinistra e destra di f in un punto x_0;
\left.f\right|_I restrizione di una funzione f all’insieme I;
\max f, \min f rispettivamente massimo e minimo della funzione f;
\sup f, \inf f rispettivamente estremo superiore ed estremo inferiore della funzione f;
A^{\circ} interno di un insieme A.


 
 

Prime definizioni e proprietà

Leggi...

In questa sezione enunciamo le prime definizioni e alcuni risultati elementari sulle funzioni convesse (e concave), analizzandone le prime proprietà ed il loro comportamento rispetto alle operazioni elementari tra funzioni.

Definizione 1.1 (funzioni convesse e concave). Sia I\subseteq \mathbb{R} un intervallo e sia f\colon I \to \mathbb{R} una funzione. Si dice che f è

    \[\quad\]

  • convessa se per ogni x_1, x_2 \in I e ogni \alpha \in [0,1] vale che

    (1)   \begin{equation*} 				f(\alpha x_1 + (1-\alpha) x_2) \leq \alpha f(x_1) + (1-\alpha) f(x_2); 			\end{equation*}

  •  

  • concava se per ogni x_1, x_2 \in I e ogni \alpha \in [0,1] vale che

    (2)   \begin{equation*} 				f(\alpha x_1 + (1-\alpha) x_2) \geq \alpha f(x_1) + (1-\alpha) f(x_2). 			\end{equation*}

Inoltre, se la diseguaglianza (1) (risp. (2)) è verificata in senso stretto, f si dice strettamente convessa (risp. strettamente concava).

Se J \subseteq I è un intervallo, f si dice convessa (risp. concava) su J se la restrizione f|_J di f a J è convessa (risp. concava).

    \[\quad\]

Da un punto di vista geometrico, la nozione di concavità e di convessità è ispirata dalla posizione reciproca tra il grafico della funzione e il segmento congiungente due punti della funzione stessa.

In particolare, in una funzione convessa il segmento congiungente due punti del grafico è sempre superiore al grafico stesso, mentre in una funzione concava il segmento è sempre inferiore. Infatti, il secondo membro della disuguaglianza (1) (risp. (2)) è l’ordinata del punto del segmento congiungente i punti (x_1,f(x_1)) e (x_2,f(x_2)) avente ascissa pari a \alpha x_1 + (1-\alpha)x_2, mentre il primo membro rappresenta l’ordinata di un punto del grafico avente ascissa pari a \alpha x_1 + (1-\alpha)x_2. Si veda la figura 1a per una funzione convessa e la figura 2 per una funzione concava.

    \[\quad\]

    \[\quad\]

Figura 1

    \[\quad\]

    \[\quad\]

Ancora, nei casi in cui abbia senso parlare di tangente al grafico (ovvero nei casi di funzioni derivabili) la retta tangente al grafico è sempre inferiore al grafico stesso nel caso di funzioni convesse, mentre è superiore al grafico per funzioni concave. Questa proprietà, che non risulta immediata dalla definizione, verrà dimostrata in seguito (teorema 2.9). Si veda figura 1b.

    \[\quad\]

    \[\quad\]

Figura 2: il grafico di una funzione concava e il segmento congiungente i punti A e B del grafico. Il segmento è al di sotto del grafico mentre la tangente sopra.

    \[\quad\]

    \[\quad\]

Funzioni concave e convesse sono strettamente legate tra di loro. In particolare, vale il seguente risultato.

Proposizione 1.2. Sia I\subseteq \mathbb{R} un intervallo e sia f\colon I \to \mathbb{R} una funzione. Allora, f è convessa se e solo se -f è concava.

    \[\quad\]

Dimostrazione. Siano x_1, x_2 \in I e \alpha \in [0,1]. Allora vale che

(3)   \begin{equation*} \begin{split} 		f(\alpha x_1 + (1-\alpha) x_2) 		\leq & 		\alpha f(x_1) + (1-\alpha) f(x_2) 		\iff\\[3pt] 		 -f(\alpha x_1 + (1-\alpha) x_2) 		 \geq& 		 -\alpha f(x_1) - (1-\alpha) f(x_2) 		= \alpha(-f(x_1)) + (1 - \alpha)(-f(x_2)). \end{split} 	\end{equation*}

In virtù della proposizione 1.2, d’ora in avanti ci concentreremo soltanto sulle funzioni convesse, in quanto tutti i risultati varranno anche per le funzioni concave, a meno di invertire il segno.

Osservazione 1.3. Nella definizione 1.1 di funzione convessa (risp. concava) non sono richieste né derivabilità né continuità della funzione f.

Esempio 1.4. In figura 3 è mostrato il grafico delle funzione valore assoluto f \colon \mathbb{R} \to \mathbb{R} definita da f(x) \coloneqq  | x | per ogni x \in \mathbb{R}. Questa funzione non è derivabile in 0, ma è convessa; infatti, se x_1, x_2 \in \mathbb{R} e \alpha \in [0,1] si ha:

(4)   \begin{equation*} \begin{split} 		f(\alpha x_1 + (1-\alpha)x_2) =& | \alpha x_1 + (1-\alpha)x_2| \\ 		 \le& | \alpha x_1 |+| (1-\alpha)x_2|\\ 		= & \alpha |x_1| + (1-\alpha)|x_2| 		\\ 		= & 		\alpha f(x_1)+(1-\alpha)f(x_2), \end{split} 	\end{equation*}

dove nella prima disuguaglinza si è applicata la disuguaglianza triangolare.

    \[\quad\]

    \[\quad\]

Figura 3: il grafico del valore assoluto. Tale funzione è convessa ma non è derivabile in 0.

    \[\quad\]

    \[\quad\]

Ci si chiede se una funzione possa essere allo stesso tempo sia concava che convessa. La risposta è affermativa: le funzioni affini sono sia concave che convesse e tali funzioni sono le uniche a possedere questa proprietà, come mostra il seguente risultato.

Proposizione 1.5. Sia f \colon \mathbb{R} \to \mathbb{R} una funzione concava e convessa. Allora f è affine.

    \[\quad\]

Dimostrazione. Dalle definizioni di funzione concava (1) e convessa (2), per ogni x_1, x_2 \in I e per ogni \alpha \in [0,1] si ha che

    \[\begin{cases} 		f(\alpha x_1 + (1 - \alpha) x_2) \leq \alpha f(x_1) + (1 - \alpha) f(x_2),\\  		f(\alpha x_1 + (1 - \alpha) x_2) \geq \alpha f(x_1) + (1 - \alpha) f(x_2), 	\end{cases}\]

da cui segue che

(5)   \begin{equation*} 		f(\alpha x_1 + (1-\alpha) x_2) = \alpha f(x_1) + (1-\alpha) f(x_2). 	\end{equation*}

Facciamo vedere che questa condizione è equivalente alla definizione di funzione affine, i.e. esistono m,q \in \mathbb{R} tali che f(x)=mx + q per ogni x \in \mathbb{R}.

Sia f una funzione tale che f(x) = mx + q per ogni x\in \mathbb{R}, con m,q \in \mathbb{R}, e siano x_1, x_2 \in \mathbb{R} e \alpha \in [0, 1]. Allora

(6)   \begin{equation*} \begin{split} 		f\left(\alpha x_1 + (1-\alpha)x_2\right) & = m(\alpha x_1 + (1 - \alpha) x_2) + q \\ 		& = \alpha m x_1 + (1 - \alpha) m x_2 + \alpha q + (1 - \alpha) q \\ 		& = \alpha (m x_1 + q) + (1 - \alpha) (mx_2 + q) \\ 		& = \alpha f(x_1) + (1 - \alpha)f(x_2). \end{split} 	\end{equation*}

Viceversa, supponiamo che valga (5) per ogni x_1,x_2 \in I e per ogni \alpha \in [0,1], e mostriamo che esistono m,q \in \mathbb{R} tali che f(x) = mx + q per ogni x \in \mathbb{R}. Si scelga [a,b] \subseteq I con a<b. È immediato verificare che

(7)   \begin{equation*} 		x 		= 		\frac{b-x}{b-a} a + \frac{x-a}{b-a} b 		\qquad 		\forall x \in [a,b]. 	\end{equation*}

Si ha \dfrac{x-a}{b-a} = 1 - \dfrac{b-x}{b-a} e dal fatto che x \in [a,b] segue che \dfrac{b-x}{b-a} \in [0,1]. Vale dunque

(8)   \begin{equation*} 		f(x) 		= 		f \left (\frac{b-x}{b-a} a + \frac{x-a}{b-a} b \right ) 		= 		\frac{b-x}{b-a} f(a) + \frac{x-a}{b-a} f(b), \end{equation*}

dove nella seconda uguaglianza abbiamo applicato (5) con x_1=a, x_2=b, \alpha=\dfrac{b-x}{b-a}. Svolgendo i calcoli abbiamo quindi

(9)   \begin{equation*} 		f(x) 		= 		\frac{f(b)-f(a)}{b-a} x + \frac{f(a)b - f(b)a}{b-a} 		\qquad 		\forall x \in [a,b], 	\end{equation*}

ossia f è affine su [a,b]. Rimane solo da mostrare che (9) è valida per ogni x_0 \in I. Se x_0 \in I, consideriamo [c,d] \subseteq I contenente x_0 e [a,b]. Per lo stesso ragionamento di sopra, f è affine su [c,d] e deve coincidere con (9) su [a,b]. Poiché due funzioni affini che coincidono su un intervallo sono uguali, f soddisfa (9) anche in x_0, quindi è affine su I.

Esempio 1.6. In figura 4 è mostrato il grafico di una funzione affine, costituito da una retta, che è una funzione sia concava che convessa.

    \[\quad\]

    \[\quad\]

Figura 4: il grafico del valore assoluto. Tale funzione è convessa ma non è derivabile in 0.

    \[\quad\]

    \[\quad\]

L’insieme delle funzioni convesse è chiuso rispetto alla somma, alla moltiplicazione per scalari non negativi e, sotto ipotesi aggiuntive, rispetto alla composizione di funzioni. Più formalmente, vale il seguente risultato.

Proposizione 1.7 (Operazioni tra funzioni convesse). Siano I,J\subseteq \mathbb{R} intervalli e siano f, g \colon I \to \mathbb{R} e h \colon J \to \mathbb{R} funzioni convesse e sia \lambda \geq 0. Allora

    \[\quad\]

  1. f + g \colon I \to \mathbb{R} è una funzione convessa;
  2.  

  3. \lambda f  \colon I \to \mathbb{R} è una funzione convessa;
  4.  

  5. se h è monotona crescente e f(I) \subseteq J, h \circ f  \colon I \to \mathbb{R} è una funzione convessa;
  6.  

  7. se f,g sono inoltre non negative e con la stessa monotonia, allora fg  \colon I \to \mathbb{R} è una funzione convessa.

    \[\quad\]

Dimostrazione.

  1. Presi x_1, x_2 \in I, \alpha \in [0,1], vale che

    (10)   \begin{equation*} \begin{split} 			(f + g)(\alpha x_1 + (1 - \alpha) x_2 ) & = f(\alpha x_1 + (1 - \alpha) x_2 ) + g(\alpha x_1 + (1 - \alpha) x_2 )  \\ 			& \leq \alpha f(x_1) + (1 - \alpha) f(x_2) + \alpha g(x_1) + (1 - \alpha) g(x_2)  \\ 			& = \alpha (f(x_1) + g(x_1)) + (1 - \alpha) (f(x_2) + g(x_2))  \\ 			& = \alpha (f + g)(x_1) + (1 - \alpha) (f + g)(x_2). \end{split} 		\end{equation*}

  2.  

  3. Segue direttamente dalle proprietà delle disequazioni.
  4.  

  5. Presi x_1, x_2 \in I, \alpha \in [0,1], vale che

    (11)   \begin{equation*} \begin{split} 			(h \circ f)(\alpha x_1 + (1 - \alpha) x_2) & = h(f(\alpha x_1 + (1 - \alpha) x_2 )) \\ 			& \leq h(\alpha f(x_1) + (1 - \alpha) f(x_2))  \\ 			& \leq \alpha h(f(x_1)) + (1 - \alpha) h(f(x_2))  \\ 			& = \alpha (h \circ f) (x_1) + (1 - \alpha) (h \circ f) (x_2), \end{split} 		\end{equation*}

    dove la prima disuguaglianza segue dal fatto che h è crescente e f convessa mentre la seconda disuguaglianza segue dalla convessità di h.

  6.  

  7. Per semplicità di notazione, presi x_1, x_2 \in I, \alpha \in [0,1], definiamo

        \[ 		\Delta \coloneqq \alpha(fg)(x_1) + (1-\alpha)(fg)(x_2) - (fg)(\alpha x_1 + (1-\alpha)x_2). 		\]

    Dimostrare la tesi è equivalente a dimostrare che \Delta \geq 0. Poiché f, g sono non negative e convesse, si ha che

        \[ 		(fg)(\alpha x_1 + (1 - \alpha)x_2) \leq \left(\alpha f(x_1) + (1-\alpha)f(x_2)\right)\left(\alpha g(x_1) + (1-\alpha)g(x_2)\right). 		\]

    Sostituendo nell’equazione di \Delta otteniamo che

    (12)   \begin{equation*} \begin{split} 			\Delta  \geq& \alpha(fg)(x_1) + (1 - \alpha)(fg)(x_2) - \left(\alpha f(x_1) + (1-\alpha)f(x_2)\right)\left(\alpha g(x_1) + (1-\alpha)g(x_2)\right) \\ 			 =& \alpha(fg)(x_1) + (1-\alpha)(fg)(x_2) - \alpha^2(fg)(x_1)+\\ 			& -(1-\alpha)^2(fg)(x_2) - \alpha(1-\alpha)f(x_1)g(x_2) - \alpha(1-\alpha)f(x_2)g(x_1) \\ 			 =& \alpha(1-\alpha)(fg)(x_1) + (1-\alpha)(1-(1-\alpha))(fg)(x_2)  - \alpha(1-\alpha)f(x_1)g(x_2) +\\ 			 & - \alpha(1-\alpha)f(x_2)g(x_1) \\ 			=& \alpha(1-\alpha)\left(f(x_1) g(x_1) + f(x_2)g(x_2) -f(x_1)g(x_2) - f(x_2)g(x_1)\right) \\ 			= &\alpha(1-\alpha)\left(f(x_1) - f(x_2)\right)\left(g(x_1) - g(x_2)\right). \end{split} 		\end{equation*}

    Poiché \alpha(1-\alpha)\geq 0 e f,g hanno la stessa monotonia, il segno di \left(f(x_1) - f(x_2)\right) e di \left(g(x_1) - g(x_2)\right) è lo stesso, per cui il prodotto delle due funzioni è positivo e \Delta \geq 0.


 
 

Caratterizzazione della convessità

Introduzione.

l fine di stabilire se una funzione è convessa, è utile avere a disposizione delle condizioni sufficienti per la convessità che siano facilmente verificabili. Questa sezione dunque è dedicata alla descrizione di condizioni equivalenti alla definizione di convessità (1).

Prima di enunciare il teorema di caratterizzazione richiamiamo la definizione di epigrafico di una funzione, cioè l’insieme dei punti del piano che stanno sopra al grafico di f.

Definizione 2.1 (epigrafico). Sia I \subseteq \mathbb{R} un intervallo e sia f\colon I \to \mathbb{R} una funzione. L’epigrafico di f è l’insieme dei punti del piano definito da

    \[E_f \coloneqq \left\{(x,y) \in I \times \mathbb{R}\; :\; y \geq f(x) \right\}.\]

    \[\quad\]

Inseriamo inoltre la definizione di insieme convesso nello spazio euclideo \mathbb{R}^n.

Definizione 2.2 (insieme convesso). Un insieme A \subseteq \mathbb{R}^n si dice convesso se per ogni coppia di punti x, y \in A il segmento che collega x a y è interamente contenuto in A, i.e.

    \begin{equation*} 				\alpha x + (1 - \alpha )y \in A \qquad \forall x,y \in A, \,\, \forall \alpha \in [0,1]. 			\end{equation*}

Un insieme A \subseteq \mathbb{R}^n si dice strettamente convesso se

(13)   \begin{equation*} \alpha x +(1-\alpha)y \in A^{\mathrm{o}} \qquad \forall x,y \in A, \,\, \forall \alpha \in (0,1). \end{equation*}

    \[\quad\]

Enunciamo ora il teorema di caratterizzazione della convessità per funzioni qualsiasi.

Teorema 2.3 (caratterizzazione della convessità). Sia I \subseteq \mathbb{R} un intervallo e sia f \colon I \to \mathbb{R} una funzione. Allora le seguenti proprietà sono equivalenti:

  1. f è (strettamente) convessa;
  2. se x_1,x_2 \in I con x_1 < x_2, si ha

    (14)   \begin{equation*} 				f(x) \underset{(\text{risp.} <)}{\leq} 				f(x_1) + \frac{f(x_2)-f(x_1)}{x_2-x_1} (x-x_1) 				\qquad 				\forall x \in (x_1,x_2); 			\end{equation*}

  3. l’epigrafico di f è (strettamente) convesso in \mathbb{R}^2;
  4. I rapporti incrementali di f sono (strettamente) crescenti:

    (15)   \begin{equation*} 					\frac{f(x_2)-f(x_1)}{x_2-x_1} 					 \underset{(\text{risp.} <)}{\leq} 					\frac{f(x_3)-f(x_1)}{x_3-x_1} 				 \underset{(\text{risp.} <)}{\leq} 					\frac{f(x_3)-f(x_2)}{x_3-x_2}, \qquad \forall x_1 < x_2 < x_3 \in I. 			\end{equation*}

    \[\quad\]

Dimostrazione. Proviamo che 1 \Rightarrow 2, 2 \Rightarrow 3, 3 \Rightarrow 4 e che 4 \Rightarrow 1.

    \[\quad\]

  • 1 \Rightarrow 2. Siano x_1,x_2 \in I con x_1 < x_2 e sia x \in (x_1,x_2). Si ha

    (16)   \begin{equation*} 			x= \frac{x_2-x}{x_2-x_1} x_1 + \frac{x-x_1}{x_2-x_1} x_2  		\end{equation*}

    con

    (17)   \begin{equation*} 			\frac{x-x_1}{x_2-x_1} = 1 - \frac{x_2-x}{x_2-x_1} 			\quad 			\text{e inoltre} 			\quad 			\frac{x_2-x}{x_2-x_1} \in (0,1) 		\end{equation*}

    poiché x_1 < x < x_2. Dunque vale

    (18)   \begin{equation*} \begin{split} 				f(x) 				= & 				f \left ( \frac{x_2-x}{x_2-x_1} x_1 + \frac{x-x_1}{x_2-x_1} x_2 \right ) 				\\[4pt] 				\leq & 				\frac{x_2-x}{x_2-x_1} f(x_1) 				+  				\frac{x-x_1}{x_2-x_1} f(x_2) 				\\[4pt] 				= & 				\frac{x_2- x_1 + x_1-x}{x_2-x_1} f(x_1) 				+  				\frac{x-x_1}{x_2-x_1} f(x_2) 				\\[4pt] 				= & 				f(x_1) + 				\frac{x_1-x}{x_2-x_1} f(x_1) 				+  				\frac{x-x_1}{x_2-x_1} f(x_2) \end{split} 		\end{equation*}

    dove nella disuguaglianza abbiamo usato (1). Riarrangiando i termini si ottiene (14).

  •  

  • 2 \Rightarrow 3. Mostriamo che, dati P_1=(x_1,y_1), P_2=(x_2,y_2) \in E_f con x_1 < x_2, allora tutto il segmento congiungente P_1 e P_2 è contenuto in E_f. Dalla geometria analitica elementare, un punto (x,y) di tale segmento è tale che x \in (x_1,x_2) e soddisfa l’equazione

    (19)   \begin{equation*} 			y= y_1 + \frac{x-x_1}{x_2-x_1} y_2 			\geq 			f(x_1) + \frac{x-x_1}{x_2-x_1} f(x_2) 			\geq 			f(x), 		\end{equation*}

    dove la prima disuguaglianza segue da y_1 \geq f(x_1), y_2 \geq f(x_2) poiché P_1,P_2 \in E_f, mentre la seconda applicando (14). Ciò mostra che (x,y) \in E_f e dunque l’epigrafico è convesso.

  •  

  • 3 \Rightarrow 4. Mostriamo solo la prima delle due disuguaglianze in (15), in quanto l’altra si prova in maniera analoga. Siano quindi x_1,x_2,x_3 \in I con x_1 < x_2 <x_3, definiamo le quantità

    (20)   \begin{equation*} 			m_2 \coloneqq \frac{f(x_2)- f(x_1)}{x_2 - x_1}, 			\qquad 			m_3 \coloneqq \frac{f(x_3)- f(x_1)}{x_3 - x_1} 		\end{equation*}

    e dimostriamo che m_2 \leq m_3.

    Osserviamo che m_2 è il coefficiente angolare della retta passante per i punti del piano (x_1,f(x_1)) e (x_2,f(x_2)), e che m_3.è il coefficiente angolare della retta passante per i punti del piano (x_1,f(x_1)) e (x_3,f(x_3)), aventi rispettivamente equazioni

    (21)   \begin{equation*} 			g_{2}(x) = f(x_1) + m_2(x-x_1), 			\quad 			g_{3}(x) = f(x_1) + m_3(x-x_1) 			\qquad 			\forall x \in \mathbb{R}. 		\end{equation*}

    Da queste equazioni si vede che m_2 \leq m_3 se e solo se esiste x > x_1 tale che g_2(x) \leq g_3(x). Dimostreremo che g_2(x_2) \leq g_3(x_2).

    Poiché il grafico di g_3 contiene il segmento congiungente i punti del piano (x_1,f(x_1)) e (x_3,f(x_3)), poiché tali punti sono contenuti nell’epigrafico E_f di f per definizione, e dal fatto che tale epigrafico è convesso per ipotesi, si ha che

    (22)   \begin{equation*} 			(x,g_3(x)) \in E_f 			\quad 			\forall x \in [x_1,x_3] 			\iff 			g_3(x) \geq f(x) 			\qquad 			\forall [x_1,x_3]. 		\end{equation*}

    Dato che x_2 \in [x_1,x_3], ciò implica che g_3(x_2) \geq f(x_2) = g_2(x_2) che dimostra la prima disuguaglianza in (15).

  •  

  • 4 \Rightarrow 1. Siano x_1,x_2 \in I con x_1 < x_2 e sia \alpha \in [0,1]. Dato che (1) è ovvia per \alpha=0 e \alpha=1, fissiamo \alpha \in (0,1). Si ha

    (23)   \begin{equation*} 			\frac{f \big( \alpha x_1 + (1-\alpha) x_2 \big) - f(x_1)}{(1-\alpha)(x_2-x_1)} 			= 			\frac{f \big( \alpha x_1 + (1-\alpha) x_2 \big) - f(x_1)}{\big(\alpha x_1 + (1-\alpha) x_2\big) - x_1} 			\leq 			\frac{f(x_2) - f(x_1)}{x_2 - x_1}, 		\end{equation*}

    dove la disuguaglianza segue da (15) e dal fatto che x_1 < \alpha x_1 + (1-\alpha) x_2 < x_2. Moltiplicando il primo e il terzo membro della disuguaglianza di sopra per (1-\alpha)(x_2-x_1) e svolgendo i prodotti si ottiene (1), ossia la conclusione.

Osservazione 2.4. Dal punto di vista geometrico, il punto 3 del teorema 2.3 ci dice che, presi due punti x_1,x_2\in I il grafico della funzione sta non al di sopra del grafico della corda congiungente (x_1,f(x_1)) e (x_2,f(x_2)).

Le disuguaglianze centrali in (15) derivano direttamente da quella tra il primo e il terzo membro, come mostra il prossimo lemma, che utilizzeremo nel seguito e che è interessante di per sé.

Lemma 2.5. Sia I \subseteq \mathbb{R} un intervallo, sia f \colon I \to \mathbb{R} una funzione e siano x_1 < x_2 < x_3 \in I. Se vale

(24)   \begin{equation*} 			\frac{f(x_2)-f(x_1)}{x_2-x_1} 			\leq 			\frac{f(x_3)-f(x_2)}{x_3-x_2}, 		\end{equation*}

allora si ha

(25)   \begin{equation*} 			\frac{f(x_2)-f(x_1)}{x_2-x_1} 			\leq 			\frac{f(x_3)-f(x_1)}{x_3-x_1} 			\leq 			\frac{f(x_3)-f(x_2)}{x_3-x_2}. 		\end{equation*}

    \[\quad\]

Dimostrazione. Chiamiamo

(26)   \begin{equation*} 		m_{12} \coloneqq 		\frac{f(x_2)-f(x_1)}{x_2-x_1}, 		\qquad 		m_{13} \coloneqq 		\frac{f(x_3)-f(x_1)}{x_3-x_1}, 		\qquad 		m_{23} \coloneqq 		\frac{f(x_3)-f(x_2)}{x_3-x_2}, 	\end{equation*}

e supponiamo di sapere che m_{12} \leq m_{23}. La conclusione equivale a dimostrare che m_{12} \leq m_{13} \leq m_{23}. Con semplici calcoli algebrici si ha

(27)   \begin{equation*} 		\begin{split} 			m_{13} (x_3 - x_2) + m_{13}(x_2 - x_1) 			= & 			m_{13} (x_3 - x_1) 			\\ 			= & 			f(x_3) - f(x_1) 			\\ 			= 			& 			f(x_3) - f(x_2) + f(x_2) - f(x_1) 			\\ 			= & 			m_{23} (x_3-x_2) + m_{12}(x_2-x_1). 		\end{split} 	\end{equation*}

Confrontando il primo e l’ultimo membro si ottiene

(28)   \begin{equation*} 		(m_{13} - m_{12})(x_2 - x_1) 		= 		(m_{23} - m_{13})(x_3 - x_2). 	\end{equation*}

Dato che i fattori (x_2 - x_1) e (x_3 - x_2) sono positivi, l’uguaglianza in (28) implica che i numeri m_{13} - m_{12} e m_{23} - m_{13} devono avere lo stesso segno. Dato che la loro somma è pari a m_{23} - m_{12} che è non-negativa per ipotesi, si ha che

(29)   \begin{equation*} 		m_{13} - m_{12} \geq 0, 		\qquad 		m_{23} - m_{13} \geq 0, 	\end{equation*}

ossia

(30)   \begin{equation*} 		m_{12} \leq m_{13} \leq m_{23}, 	\end{equation*}

che è la tesi.

Osservazione 2.6. Il lemma 2.5 ha un’interessante interpretazione geometrica, rappresentata in figura 5: chiamando

(31)   \begin{equation*} 		P_1 \coloneqq \big(x_1, f(x_1)\big), 		\qquad 		P_2 \coloneqq \big(x_2, f(x_2)\big), 		\qquad 		P_3 \coloneqq \big(x_3, f(x_3)\big), 	\end{equation*}

la pendenza della retta r_{13} congiungente P_1 e P_3 è in un certo senso “una media” delle pendenze delle rette r_{12} e r_{23} congiungenti rispettivamente P_1 con P_2 e P_2 con P_3.

    \[\quad\]

    \[\quad\]

Figura 5: rappresentazione grafica del lemma 2.5; il coefficiente angolare della retta r_{13} è compreso tra i coefficienti angolari delle rette r_{12} e r_{23}.

    \[\quad\]

    \[\quad\]

Enunciamo ora un risultato di convessità per cui è necessaria l’ipotesi di continuità della funzione in esame. Tale proposizione afferma che, se una funzione è continua, allora per mostrarne la convessità è sufficiente verificare la disuguaglianza di convessità soltanto nel punto medio del segmento considerato, ossia per \alpha= \dfrac{1}{2}.

Teorema 2.7. Sia I\subseteq \mathbb{R} un intervallo e sia f\colon I \to \mathbb{R} una funzione continua e tale che

(32)   \begin{equation*} 		f\left(\frac{x_1+x_2}{2}\right) \leq \frac{f(x_1) + f(x_2)}{2} \qquad \forall x_1,x_2 \in I. 	\end{equation*}

Allora f è convessa.

    \[\quad\]

Dimostrazione. Supponiamo per assurdo che f sia continua e che soddisfi (32), ma che non sia convessa nel senso della definizione 1.1. Per il punto 4 e che 4 del teorema 2.3, esistono quindi a,x,b \in I con a <x< b tali che

    \[f(x) > \dfrac{f(b)-f(a)}{b-a} (x - a)-f(a).\]

Definendo quindi la funzione g \colon [a,b] \to \mathbb{R} come

    \[g(t) \coloneqq f(t) - \dfrac{f(b)-f(a)}{b-a} (t - a)-f(a) \qquad \forall t \in [a,b],\]

ciò è equivalente ad assumere che g(x)>0. Mostriamo che ciò conduce a una contraddizione.

Poiché f è continua, anche la funzione g è continua; inoltre, poiché essa è definita in un intervallo chiuso e limitato, ammette massimo e minimo per il teorema di Weierstrass [3, teorema 5.30]. Dunque esiste M \coloneqq \max_{x\in [a,b]} g(x) e, poiché g(x)>0, segue che M>0.

Osserviamo che, poiché f soddisfa (32), anche g soddisfa (32); infatti se x_1, x_2 \in [a,b], allora

(33)   \begin{equation*} \begin{split} 		g\left(\frac{x_1+x_2}{2}\right) = & f\left(\frac{x_1+x_2}{2}\right) - \dfrac{f(b)-f(a)}{b-a} \left(\frac{x_1+x_2}{2} - a\right)-f(a)\\ 		\leq & \frac{f(x_1) + f(x_2)}{2} - \dfrac{f(b)-f(a)}{b-a} \left(\frac{x_1+x_2}{2} - a\right)-f(a)\\ 		= & \dfrac{1}{2}\left(f(x_1) + f(x_2) -  \dfrac{f(b)-f(a)}{b-a} (x_1 + x_2 - 2a) - 2f(a)  \right)\\ 		= &  \dfrac{1}{2}\left(f(x_1)-  \dfrac{f(b)-f(a)}{b-a} (x_1 - a) - f(a)  + f(x_2)- \dfrac{f(b)-f(a)}{b-a} (x_2 - a) - f(a) \right)\\ 		= & \dfrac{g(x_1) + g(x_2)}{2}. \end{split} 	\end{equation*}

Sia \bar{x} \coloneqq \inf\{x \in [a,b] \colon g(\bar{x}) = M \}; sia \bar{x}_k una successione tale che g(\bar{x}_k)=M e tale che \bar{x}_k \to \bar{x}. Per la continuità di g e per la caratterizzazione della continuità per successioni [3, teorema 3.3], allora

(34)   \begin{equation*} 			g(\bar{x})=M. 	\end{equation*}

Inoltre segue che \bar{x}\neq a e \bar{x} \neq b (altrimenti g(a)=M o g(b)=M, che è assurdo poichè g(a)=g(b) = 0 e M>0). Dunque si ha che \bar{x} \in (a,b), da cui esiste \delta >0 tale che (\bar{x}-\delta, \bar{x}+\delta)\subset [a,b]. Allora si ha che

    \[2M = 2g(\bar{x}) \leq  g(\bar{x} - \delta) + g(\bar{x}+\delta) < M + M,\]

dove la prima disuguaglianza segue dalla convessità nel punto medio e la disuguaglianza g(x-\delta)< M segue dal fatto che \bar{x}-\delta < \bar{x}, che è l’estremo inferiore dei punti di massimo assoluti per g, mentre g(\bar{x}+\delta) \leq M poiché M=\max g.

Quindi abbiamo appena verificato che

    \[ 	g(\bar{x}) < M, 	\]

che contraddice (34), da cui l’assurdo.

Osservazione 2.8. Senza l’ipotesi di continuità, una funzione convessa nel punto medio non è necessariamente convessa. Dei controesempi possono essere costruiti con l’uso dell’Assioma della Scelta, tuttavia ciò esula dalla trattazione di questa dispensa.


Caratterizzazione delle funzioni convesse derivabili.

Per le funzioni derivabili una o due volte, possiamo dare ulteriori caratterizzazioni della convessità.

Finora abbiamo enunciato un teorema di caratterizzazione delle funzioni convesse (teorema 2.3), mentre nel teorema 2.7 abbiamo provato un criterio di convessità per funzioni continue. Enunciamo ora un ulteriore teorema di caratterizzazione, nel quale si richiede che la funzione in esame sia anche derivabile.

Teorema 2.9 (caratterizzazione delle funzioni convesse derivabili). Sia I \subseteq \mathbb{R} un intervallo e sia f\colon I \to \mathbb{R} una funzione continua in I e derivabile in I^{\circ}. Allora le seguenti proprietà sono equivalenti:

  1. f è (strettamente) convessa;
  2. per ogni x_1 \in I^{\circ}, x_2 \in I si ha che

        \[f(x_2) \geq f(x_1) + f'(x_1)(x_2-x_1) \qquad \text{(risp. $f(x_2) > f(x_1) + f'(x_1)(x_2-x_1) $)}.\]

  3. f' è una funzione (strettamente) crescente in I^{\circ}.

    \[\quad\]

Dimostrazione. Proviamo che 1 \Rightarrow 2, 2 \Rightarrow 3 e che 3 \Rightarrow 1.

    \[\quad\]

  • 1 \Rightarrow 2. Poiché f è convessa, vale il punto 3 del teorema 2.3. Fissiamo x_1,x_2 \in I^{\mathrm{o}} e supponiamo che x_1<x_2, in quanto l’altro caso è analogo. Si ha

        \[f(x) \leq f(x_1) + \dfrac{f(x_2)-f(x_1)}{x_2-x_1}(x-x_1) \qquad \forall x \in (x_1,x_2).\]

    Tale disuguaglianza si può scrivere come

        \[\dfrac{f(x)-f(x_1)}{x-x_1}(x_2-x_1) \leq  f(x_2)-f(x_1)  \qquad \forall x \in (x_1,x_2),\]

    Poiché tale disuguaglianza rimane vera effettuando il limite per x \to x_1, per la derivabilità di f in x_1 si ottiene

        \[f'(x_1) (x_2-x_1) \leq f(x_2) - f(x_1),\]

    ossia la tesi.

  •  

  • 2 \Rightarrow 3. Siano x_1,x_2 \in I^{\circ} con x_1<x_2 e proviamo che f'(x_1) \leq f'(x_2). Per ipotesi sappiamo che

        \[f'(x_1) \leq \dfrac{f(x_2)-f(x_1)}{x_2-x_1}.\]

    Allo stesso modo, scambiando i ruoli di x_1 ed x_2, si ha che

        \[f'(x_2) \leq \dfrac{f(x_1)- f(x_2)}{x_1-x_2},\]

    da cui si ha la tesi mettendo assieme le due disuguaglianze.

  •  

  • 3 \Rightarrow 1. Dimostreremo che vale il punto 4 del teorema 2.3, quindi consideriamo x_1, x_2 < x_3 \in I. Per il teorema di Lagrange applicato agli intervalli [x_1,x_2] e [x_2,x_3], esistono a \in (x_1,x_2) e b \in (x_2,x_3) tali che

    (35)   \begin{equation*}	 				\frac{f(x_2)- f(x_1)}{x_2 - x_1} 				= 				f'(a) 				\leq 				f'(b) 				= 				\frac{f(x_3) - f(x_2)}{x_3 - x_2}, 			\end{equation*}

    dove la disuguaglianza segue dall’ipotesi di monotonia di f' in quanto per costruzione si ha a < b. Da ciò e dal lemma 2.2, si ha

    (36)   \begin{equation*} 				\frac{f(x_2)- f(x_1)}{x_2 - x_1} 				\leq 				\frac{f(x_3)- f(x_1)}{x_3 - x_1} 				\leq 				\frac{f(x_3) - f(x_2)}{x_3 - x_2}, 			\end{equation*}

    ossia la conclusione.

Enunciamo ora un’ulteriore caratterizzazione della convessità, nel caso in cui f sia derivabile due volte all’interno del suo intervallo di definizione.

Teorema 2.10. Sia I \subseteq \mathbb{R} un intervallo e sia f\colon I \to \mathbb{R} una funzione continua in I e derivabile due volte in I^{\circ}. Allora f è convessa se e solo se f''(x) \ge 0 per ogni x \in I^{\circ}.

    \[\quad\]

Dimostrazione. Se f è convessa, poiché f è derivabile in I^{\circ} per il teorema v allora f' è crescente; poiché f' è a sua volta derivabile in I^{\circ} ed è crescente, allora f'' \ge 0 in I^{\circ}. Viceversa, Se f''\ge 0 in I^{\circ} allora f' è crescente in in I^{\circ}; applicando il teorema 2.9 segue che f è convessa.


 
 

Convessità, continuità e derivabilità

Introduzione.

In questa sezione esploreremo il rapporto tra funzioni convesse e la loro continuità e derivabilità.

Legame tra continuità e convessità.

Nell’osservazione 1.3 abbiamo precisato che affinché una funzione f sia convessa non è necessario che essa sia derivabile o continua. Ci si chiede dunque cosa leghi il concetto di convessità a quello di continuità.

Partiamo da un concetto più debole di continuità, ossia la semicontinuità superiore (risp. inferiore).

Definizione 3.1 (funzione semicontinua superiormente o inferiormente). Sia I \subseteq \mathbb{R} un intervallo e sia x_0\in I. Una funzione f \colon I \to \mathbb{R} si dice semicontinua superiormente (risp. inferiormente) in x_0 se

    \[\limsup_{x \to x_0} f(x) \leq f(x_0) \qquad \text{(risp. $\liminf_{x \to x_0} f(x) \geq f(x_0)$)}.\]

Proposizione 3.2 (semicontinuità delle funzioni convesse). Sia I \subseteq \mathbb{R} un intervallo e sia f \colon I \to \mathbb{R} una funzione convessa. Allora f è semicontinua superiormente.

    \[\quad\]

Dimostrazione. Fissato x_1 \in I, vogliamo mostrare che f è semicontinua superiormente in x_1; scegliendo \varepsilon>0, per la definizione di \limsup occorre e basta dimostrare che esiste \delta>0 tale che

(37)   \begin{equation*} 		f(x) 		\leq 		f(x_1) + \varepsilon 		\qquad 		\forall x \in (x-\delta,x+\delta). 	\end{equation*}

Scegliamo x_2 \in I con x_2 \neq x_1 e supponiamo per fissare le idee che x_1 < x_2. Per la convessità di f e per (14) si ha

(38)   \begin{equation*} 		f(x) \leq f(x_1) + \frac{f(x_2)-f(x_1)}{x_2-x_1}(x-x_1) 		\eqqcolon 		g(x) 		\qquad 		\forall x \in (x_1,x_2), 	\end{equation*}

dove quindi g \colon \mathbb{R} \to \mathbb{R} è una funzione affine. Poiché g è continua, esiste \delta>0 tale che

(39)   \begin{equation*} 		g(x)< g(x_1) + \varepsilon 		\qquad 		\forall x \in (x_1-\delta,x_1+\delta). 	\end{equation*}

Unendo queste considerazioni si ha dunque

(40)   \begin{equation*} 		f(x) 		\leq 		g(x) 		< g(x_1)+\varepsilon 		= 		f(x_1)+ \varepsilon 		\qquad 		\forall x \in (x_1,x_1+\delta), 	\end{equation*}

dove la prima disuguaglianza segue da (38), la seconda da (39) e l’uguaglianza da g(x_1)=f(x_1). Scegliendo d’altra parte x_2< x_1, si ottiene la medesima disuguaglianza in un intorno sinistro di x_1 e ciò quindi dimostra (37), ossia la semicontinuità superiore di f in x_1.

Ci si chiede se ci siano legami più forti con la continuità. La risposta è affermativa e nel seguito investighiamo il rapporto tra funzioni convesse e lipschitziane. Ricordiamo la definizione di funzione lipschitziana e rimandiamo a [3, sezione 6.3] per una trattazione approfondita dell’argomento.

Definizione 3.3 (funzione lipschitziana). Sia I \subseteq \mathbb{R}. Una funzione f\colon I \to \mathbb{R} è lipschitziana se esiste L\geq 0 tale che

    \[ 		|f(x) - f(y)| \leq L|x - y| \qquad \forall x, y \in I. 		\]

    \[\quad\]

Il seguente risultato afferma che una funzione convessa è localmente lipschitziana.

Teorema 3.4. Sia I \subseteq \mathbb{R} un intervallo e sia f\colon I \to \mathbb{R} una funzione convessa. Allora per ogni intervallo chiuso e limitato [c,d] \subset I^{\mathrm{o}}, f ristretta a [c,d] è lipschitziana. In particolare, f è continua in I^{\mathrm{o}}.

    \[\quad\]

Dimostrazione. Senza perdita di generalità possiamo supporre I=[a,b] e siano c,d \in \mathbb{R} tali che [c,d] \subset (a,b). Siano t_1,t_2 tali che a < t_2 < c < d <t_1 < b. Poiché f è convessa, per il punto 4 del teorema 2.3 si ha

    \[\dfrac{f(c)- f(t_2)}{c-t_2}\leq \dfrac{f(x)-f(y)}{x-y} \leq \dfrac{f(t_1)-f(d)}{t_1-d}, \qquad \forall x,y \in [c,d].\]

Sia

    \[L \coloneqq \max \left\lbrace \left|\dfrac{f(c)- f(t_2)}{c-t_2}\right|, \left|\dfrac{f(t_1)-f(d)}{t_1-d}\right|\right\rbrace .\]

allora si ha che

    \[|f(x)-f(y)| \leq L|x-y| , \qquad \forall x,y \in [c,d]\]

Ciò prova che f è lipschitziana su [c,d].

Facciamo vedere ora che f è continua in {I}^{\mathrm{o}}. Siano x_0 \in (a,b) e \delta >0 tali che

    \[[x_0 - \delta, x_0 + \delta] \subset (a,b).\]

Allora per quanto appena provato, f è lipschitziana su [x_0 - \delta, x_0 + \delta], cioè esiste L\ge 0 tale che

    \[|f(x)-f(y)| \le L |x-y| \qquad \forall x, y \in [x_0 - \delta, x_0 + \delta].\]

Come provato in [3, corollario 6.26, ogni funzione lipschitziana è uniformemente continua (e dunque continua); per completezza, proviamo questa asserzione. Fissato \varepsilon > 0, si ha

(41)   \begin{equation*} |f(x)-f(y)| \le L|x-y| < L \dfrac{\varepsilon}{L} = \varepsilon \qquad \forall x,y \in [x_0-\delta,x_0+\delta]\,\, \text{ con } \,\, |x-y|< \dfrac{\varepsilon}{L}, \end{equation*}

che prova l’uniforme continuità di f in [x_0 - \delta, x_0 + \delta] e in particolare la continuità di f in 0.

Restringersi all’interno dell’intervallo I nel teorema 3.4 è essenziale. Si consideri infatti il seguente esempio di funzione convessa su un intervallo chiuso ma non continua al bordo.

Esempio 3.5. Sia f\colon [0,1]\to \mathbb{R} definita da

    \[f(x) = \begin{cases*} 		1, & \text{se } $x=0$,\\ 		0, & \text{altrimenti}, 	\end{cases*}\]

rappresentata in figura 6.

    \[\quad\]

    \[\quad\]

Figura 6: la funzione f dell’esempio 3.5 è convessa ma è discontinua in 0. In ogni caso, f è superiormente semicontinua in 0.

    \[\quad\]

    \[\quad\]

Chiaramente f non è continua in x=0, infatti

    \[\lim_{x\to 0} f(x) = 0 \neq f(0) = 1.\]

Proviamo che f è convessa. Siano x_1, x_2 \in (0,1] e \alpha \in [0,1], allora

    \[f(\alpha x_1 + (1-\alpha )x_2) = 0 \quad \text{e} \quad \alpha  f(x_1) + (1-\alpha )f(x_2) = \alpha \cdot 0 + (1-\alpha )\cdot 0 = 0.\]

Se x_1=0 (o equivalentemente x_2=0) procedendo analogamente si ha che

    \[f(\alpha x_1 + (1-\alpha )x_2) = 0 \leq \alpha  = \alpha \cdot 1 + (1-\alpha )\cdot 0 = \alpha f(x_1) + (1-\alpha )f(x_2).\]


Legame tra derivabilità e convessità.

Dopo aver investigato il legame tra continuità e convessità, ci interroghiamo sul legame tra funzioni convesse e derivabili. Ricordiamo la caratterizzazione equivalente di convessità data dal punto 4 del teorema 2.3, i.e. f \colon I \subseteq \mathbb{R} è convessa se e solo se

    \[\frac{f(x_2) - f(x_1)}{x_2-x_1} \leq  \frac{f(x_3) - f(x_1)}{x_3-x_1} \le  \frac{f(x_3) - f(x_2)}{x_3-x_2}, \qquad \forall x_1<x_2<x_3 \in I.\]

Lemma 3.6 (derivate sinistra e destra di funzioni convesse). Sia I \subseteq \mathbb{R} un intervallo e sia f \colon I \to \mathbb{R} una funzione convessa. Allora le derivate sinistra e destra di f esistono in ogni punto di I e vale1

(42)   \begin{equation*} 			f'_-(x_1) 			\leq 			f'_+(x_1) 			\leq 			f'_-(x_2) 			\qquad 			\forall x_1,x_2 \in I\,\, \text{con } x_1 < x_2. 		\end{equation*}

Inoltre, se x \in I^{\mathrm{o}}, allora f'_-(x) e f'_+(x) sono finite.


  1. Ovviamente nell’eventuale estremo sinistro di I esiste soltanto la derivata destra f'_+, mentre nell’estremo destro di I esiste soltanto la derivata sinistra f'_-.

    \[\quad\]

Dimostrazione. Dato che il risultato è locale, dipende cioè soltanto dalle proprietà di f in un intorno del punto considerato, possiamo assumere senza perdita di generalità che l’intervallo I=[a,b] sia chiuso e limitato.

    \[\quad\]

  • Esistenza delle derivate sinistra e destra di f. Fissiamo x_1 \in I con x_1 \neq b e mostriamo che f è derivabile a destra in x_1. Poiché f è convessa, per il punto 4 del teorema 2.3 si ha

    (43)   \begin{equation*} 			\frac{f(x_2)- f(x_1)}{x_2-x_1} 			\leq 			\frac{f(x_3)- f(x_1)}{x_3-x_1} 			\qquad 			\forall x_2,x_3 \in I \,\,\text{con } x_2 < x_3. 		\end{equation*}

    Dunque la funzione

    (44)   \begin{equation*} 			x \in (x_1,b] \longmapsto \frac{f(x)-f(x_1)}{x-x_1}, 		\end{equation*}

    che a x \in (x_1,b] associa il rapporto incrementale di f nell’intervallo (x_1,x), è crescente. Grazie a questa monotonia si ha che esiste il limite

    (45)   \begin{equation*} 			f'_+(x_1) 			= 			\lim_{x \to x_1^+} \frac{f(x)-f(x_1)}{x-x_1} 			= 			\inf_{x> x_1} \frac{f(x)-f(x_1)}{x-x_1}, 		\end{equation*}

    che è proprio la derivata destra di f in x_1. In maniera analoga si mostra l’esistenza della derivata sinistra di f in x_1 se x_1 \in (a,b].

  •  

  • Vale (42). Siano x_1,x_2 \in I con x_1 < x_2. Applicando di nuovo il il punto 4 del teorema 2.3 si ha

    (46)   \begin{equation*} 			\frac{f(x_1)-f(\alpha)}{x_1 - \alpha} 			\leq 			\frac{f(\beta)-f(x_1)}{\beta - x_1} 			\leq 			\frac{f(\gamma)- f(\beta)}{\gamma - \beta} 			\leq 			\frac{f(x_2)- f(\gamma)}{x_2- \gamma} 			\qquad \text{se } a < \alpha < x_1 <\beta <\gamma < x_2. 		\end{equation*}

    Calcolando i limiti dei rapporti in (46) per \alpha \to x_1^-, per \beta \to x_1^+ e per \gamma \to x_2^-, e considerando che tali limiti esistono per quanto provato al punto precedente, si ottiene

    (47)   \begin{equation*} 			f'_-(x_1) \leq f'_+(x_1) \leq 			\frac{f(x_2)-f(x_1)}{x_2-x_1} 			\leq 			f'_-(x_2), 		\end{equation*}

    cioè la tesi.

  •  

  • f'_-(x) e f'_+(x) sono finite in I^{\mathrm{o}}.

    Fissiamo x_1 \in (a,b) e scegliamo \delta>0 tale che [x_1-\delta,x_1+\delta] \subset (a,b). Per il teorema 3.4, f è lipschitziana in [x_1-\delta,x_1+\delta], quindi da (45) segue che il valore f'_+(x_1) è limitato dalla costante di Lipschitz di f in [x_1-\delta,x_1+\delta], dunque esso è finito. In maniera analoga si mostra che f'_-(x_1) è finita.

Come immediata conseguenza del lemma precedente, otteniamo la derivabilità di una funzione convessa in tutti i punti interni, tranne al più una quantità numerabile di essi.

Teorema 3.7 (derivabilità delle funzioni convesse). Sia I \subseteq \mathbb{R} un intervallo e sia f \colon I \to \mathbb{R} una funzione convessa. Allora f è derivabile in I^{\mathrm{o}} tranne una quantità al più numerabile di punti. Inoltre f' è, quando definita, una funzione crescente.

    \[\quad\]

Dimostrazione. Per il lemma 3.6 sappiamo che in ogni punto di I^{\circ} esistono derivata destra e sinistra di f e per ogni x_0,x_1 \in I^{\circ} con x_0< x_1 si ha che

(48)   \begin{equation*} 		f'_{-}(x_0) \leq f'_{+}(x_0) \leq f'_{-}(x_1) \leq f'_{+}(x_1).	 	\end{equation*}

Consideriamo l’insieme

    \[\Delta \coloneqq \left\lbrace x \in I^{\circ} \; \colon \; f'_{-}(x)<f'_{+}(x)\right\rbrace.\]

A ciascun x\in \Delta si può associare l’intervallo non vuoto I_x \coloneqq (f'_{-}(x), f'_{+}(x)). Pertanto, ad ogni x\ \in \Delta si può associare (grazie all’assioma delle scelte numerabili) un numero razionale q_x \in \mathbb{Q} tale che

    \[f'_{-}(x)< q_x <f'_{+}(x), \qquad \forall x \in \Delta.\]

Per (48) si ha che per ogni x\in I^{\circ} gli intervalli I_x sono a due a due disgiunti e dunque se x, y \in \Delta, x\neq y si ha che q_x \neq q_y. Segue dunque che, la funzione appena definita, che associa a ogni elemento x dell’insieme \Delta un numero razionale, è iniettiva. Poiché l’insieme \mathbb{Q} è numerabile, ciò implica che \Delta è al più numerabile, cioè la tesi.

Osserviamo infine che, nei punti in cui f' esiste, essa chiaramente coincide poiché f'_- e con f'_+. Poiché ognuna di queste due funzioni è crescente, anche f' risulta crescente.

Osservazione 3.8. L’ultimo asserto sulla monotonia di f' non si poteva ricavare dal teorema 2.9, in quanto tale risultato afferma che la convessità è equivalente al fatto che f' sia crescente, sotto l’ipotesi che f' esista ovunque in I^{\mathrm{o}}. Poiché abbiamo visto che la funzione valore assoluto x \in \mathbb{R} \mapsto |x| è convessa ma non derivabile in 0, non si poteva applicare la caratterizzazione delle funzioni convesse derivabili data dal teorema 2.9 per affermare che f' sia crescente.

D’altra parte però, il teorema 3.7 fornisce un’altra dimostrazione del fatto che, se f è convessa e derivabile, allora f' è crescente.


 
 

Estremi di una funzione convessa

Leggi...

Dopo aver investigato i legami delle funzioni convesse con continuità e derivabilità, ci si chiede se tali funzioni abbiano massimi/minimi.

Cominciamo trattando i punti di minimo di funzioni convesse.

Teorema 4.1. Siano I\subseteq \mathbb{R} un intervallo e sia f \colon I \to \mathbb{R} una funzione strettamente convessa. Se esiste un punto x_0 \in I di minimo per f, allora esso è unico.

    \[\quad\]

Dimostrazione. Supponiamo per assurdo che esistano due punti di minimo x_0, x_1 per f. Dalla definizione di minimo si ha necessariamente che

    \[f(x_0) = \min\limits_{x \in I} f(x) = f(x_1) =: m.\]

Senza perdita di generalità, supponiamo x_0 < x_1 e consideriamo il segmento [x_0, x_1], \alpha \in [0, 1]. Poiché f è strettamente convessa, dalla definizione 1.1 vale che

(49)   \begin{equation*} 	\begin{split} 		f(\alpha x_0 + (1 - \alpha) x_1) & < \alpha f(x_0) + (1 - \alpha)f(x_1)  \\ 		& = \alpha m + m - \alpha m  \\ 		& = m. 	\end{split} \end{equation*}

Questo è assurdo perché m = \min\limits_{x \in I} f(x).

Abbiamo quindi provato che, se il minimo di una funzione strettamente convessa esiste, allora il punto di minimo in cui tale valore è assunto è unico.

Grazie al teorema di Fermat [7, teorema 1], i punti di minimo relativo di una funzione f \colon (a,b) \to \mathbb{R} derivabile sono da ricercare tra i punti x \in (a,b) che soddisfano l’equazione

(50)   \begin{equation*} 	f'(x)=0, \end{equation*}

ossia i cosiddetti punti stazionari. Il teorema di Fermat non si può invertire: i punti stazionari possono essere anche di massimo, oppure non essere né di massimo né di minimo. Occorre quindi stabilirne la natura con altri metodi (studio del segno di f' in un intorno della soluzione, oppure studio del segno di f'', etc…).

Se f è una funzione convessa, invece tutto questo lavoro ulteriore non è necessario: si può provare infatti che ogni punto stazionario è di minimo assoluto per f, come stabilito dal seguente teorema. In altre parole, per una funzione convessa il teorema di Fermat si può in un certo senso invertire, come rappresentato in figura 7.

Teorema 4.2. Sia f \colon (a,b) \to \mathbb{R} una funzione convessa e sia x_0 \in (a,b) un punto in cui f è derivabile e tale che f'(x_0)=0. Allora x_0 è un punto di minimo assoluto per f.

    \[\quad\]

Dimostrazione. Fissiamo x_1 \in (a,b) e mostriamo che f(x_1) \geq f(x_0). Per fissare le idee, supponiamo x_1 \in (x_0,b) in quanto l’altro caso è analogo. Poiché f è convessa, per il punto 4 del teorema 2.3 si ha

(51)   \begin{equation*} 		\frac{f(x)-f(x_0)}{x - x_0} 		\leq 		\frac{f(x_1)- f(x_0)}{x_1 - x_0} 		\qquad 		\forall x \in (x_0,x_1). 	\end{equation*}

Poiché f è derivabile in x_0 e f'(x_0)=0 per ipotesi, calcolando il limite dell’espressione di sopra per x \to x_0^+, si ottiene

(52)   \begin{equation*} 		0 		\leq 		\frac{f(x_1)- f(x_0)}{x_1 - x_0}. 	\end{equation*}

Poiché x_1>x_0, ciò implica che f(x_1) \geq f(x_0). Analogamente si mostra che la medesima disuguaglianza vale se x_1 \in (a,x_0).

Osservazione 4.3. In realtà, con un argomento analogo e simile alla dimostrazione del lemma 3.6, si poteva mostrare il seguente risultato più forte: se f \colon (a,b) \to \mathbb{R} è una funzione convessa e x_0 \in (a,b) è tale che f'_-(x_0)=0 oppure f'_+(x_0)=0, allora x_0 è di minimo assoluto per f.

    \[\quad\]

    \[\quad\]

Figura 7: invertibilità del teorema di Fermat per funzioni convesse; un eventuale punto stazionario per una funzione convessa è necessariamente di minimo assoluto per essa.

    \[\quad\]

    \[\quad\]

I risultati di sopra si possono generalizzare a funzioni convesse in intervalli chiusi per i punti sul bordo dell’intervallo, la cui dimostrazione è lasciata come facile esercizio per il lettore.

Proposizione 4.4. Sia f \colon [a,b] \to \mathbb{R} una funzione convessa. Valgono le seguenti equivalenze:

    \[\quad\]

  1. f'_-(a) \geq 0 se e solo se a è un punto di minimo assoluto per f;
  2.  

  3. f'_+(b) \leq 0 se e solo se b è un punto di minimo assoluto per f;

    \[\quad\]

Finora abbiamo stabilito dei criteri per ottenere l’esistenza di punti di estremo di funzioni convesse. Ci chiediamo ora se sia possibile determinarne “a priori” l’esistenza, ossia senza altre informazioni a parte la convessità della funzione. Come primo risultato, osserviamo che vale la seguente generalizzazione del teorema di Weierstrass [3, teorema 5.3] a funzioni semicontinue superiormente, e quindi in particolare a funzioni convesse.

Proposizione 4.5. Sia f \colon [a,b] \to \mathbb{R} una funzione semicontinua superiormente; allora f possiede massimo assoluto.

    \[\quad\]

Dimostrazione. Sia M=\sup_{[a,b]}f e sia x_n una successione di punti in [a,b] tale che

(53)   \begin{equation*} 		\lim_{n \to +\infty} f(x_n)=M. 	\end{equation*}

Tale successione esiste per definizione di estremo superiore. Poiché [a,b] è chiuso e limitato, per il teorema di Bolzano-Weierstrass, esiste una sottosuccessione x_{n_k} di x_n convergente a \bar{x} \in [a,b]. Da ciò e dalla semicontinuità superiore di f si ha quindi

(54)   \begin{equation*} 		f(\bar{x}) 		\geq 		\lim_{k \to + \infty} f(x_{n_k}) 		= 		M, 	\end{equation*}

Poiché M=\sup_{[a,b]}f, la disuguaglianza non può essere stretta, dunque f(\bar{x})=M, ossia \bar{x} è un punto di massimo assoluto per f.

Come corollario otteniamo l’esistenza del massimo assoluto per ogni funzione convessa su un intervallo chiuso e limitato. In realtà, per funzioni convesse vale un caso particolare del cosiddetto principio del massimo, estremamente importante nello studio delle equazioni differenziali alle derivate parziali: i punti di massimo di una funzione convessa non costante possono essere solo al bordo del dominio.

Proposizione 4.6 (principio del massimo per funzioni convesse). Sia f \colon [a,b] \to \mathbb{R} una funzione convessa. Allora f possiede massimo assoluto.

Inoltre, se f possiede un punto di massimo in (a,b), allora essa è costante. In particolare, gli unici punti di massimo di una funzione strettamente convessa possono essere soltanto a o b.

    \[\quad\]

Dimostrazione. Poiché f è convessa, essa è superiormente semicontinua per la proposizione 3.2, dunque a essa si può applicare la proposizione 4.6 e ottenere l’esistenza del massimo assoluto M \coloneqq \max f.

Supponiamo ora che \bar{x} \in (a,b) sia un punto di massimo interno e mostriamo che f è costante. Poiché \bar{x} è interno, siano x_1,x_2 \in [a,b] tali che x_1 < \bar{x} < x_2. Dal punto 4 del teorema 2.3 si ha

(55)   \begin{equation*} 		\frac{f(\bar{x})- f(x_1)}{\bar{x}- x_1} 		\leq 		\frac{f(x_2) - f(\bar{x})}{x_2 - \bar{x}}, 	\end{equation*}

ma dal fatto che f(\bar{x}) = \max f, deve aversi f(x_1) \leq f(\bar{x}) e f(x_2) \leq f(\bar{x}), da cui

(56)   \begin{equation*} 		\frac{f(\bar{x})- f(x_1)}{\bar{x}- x_1} 		\geq 		0 		\geq 		\frac{f(x_2) - f(\bar{x})}{x_2 - \bar{x}}. 	\end{equation*}

Confrontando (55) e (56) l’unica possibilità è che

(57)   \begin{equation*} 		\frac{f(\bar{x})- f(x_1)}{\bar{x}- x_1} 		= 		0 		= 		\frac{f(x_2) - f(\bar{x})}{x_2 - \bar{x}}, 	\end{equation*}

cioè che f(x_1)= f(\bar{x})= f(x_2)=M. Dall’arbitrarietà di x_1 e x_2, segue che f è costantemente pari a M.

Infine, se f è strettamente convessa, in particolare non può essere costante, dunque per quanto appena provato non può avere punti di massimo all’interno del dominio.

Trattiamo ora l’esistenza di punti di minimo per funzioni convesse. Chiaramente, se f \colon [a,b] \to \mathbb{R} è una funzione convessa e continua, allora \min f esiste per il teorema di Weierstrass [3, teorema 5.3]2. Notiamo esplicitamente però che in generale una funzione convessa può non possedere minimo, anche se la convessità è stretta ed essa è definita su un intervallo chiuso e limitato, come mostrato dal prossimo esempio.

Esempio 4.7. Consideriamo la funzione f \colon [0,1] \to \mathbb{R} definita da

(58)   \begin{equation*} 		f(x) 		= 		\begin{cases} 			1			& \text{se } x=0\\ 			x^2			& \text{se } x\in (0,1] 		\end{cases} 	\end{equation*}

e rappresentata in blu in figura 8. Mostriamo che essa è strettamente convessa. Infatti, essa è derivabile 2 volte in (0,1] con f''(x)=2>0 per ogni x \in (0,1], dunque è strettamente convessa in (0,1] per il teorema 2.9; inoltre, come rappresentato in figura 8, si ha

(59)   \begin{equation*} 		\begin{split} 			f\big(\alpha \cdot 0 + (1-\alpha)x_0 \big) 			= & 			(1-\alpha)^2x_0^2 			\\ 			< & 			(1-\alpha) x_0^2 			\\ 			\leq & 			\alpha \cdot 1 + (1-\alpha) x_0^2 			\\ 			= & 			\alpha f(0) + (1-\alpha) f(x_0) 			\qquad 			\forall x_0 \in (0,1],\,\,\forall \alpha \in (0,1), 		\end{split} 	\end{equation*}

dove la prima disuguaglianza segue da \alpha \in (0,1). Dunque f è strettamente convessa in [0,1] per la definizione 1.1.

Osserviamo però che f non ha minimo, in quanto

(60)   \begin{equation*} 		\inf_{[0,1]} f 		= 		0, 		\qquad 		\text{e} 		\qquad 		f(x)>0 		\quad 		\forall x \in [0,1]. 	\end{equation*}

    \[\quad\]

    \[\quad\]

Figura 8: la funzione f dell’esempio 4.7; essa è strettamente convessa, ma non assume valore minimo.

    \[\quad\]

    \[\quad\]

Osservazione 4.8. I risultati di unicità del punto di minimo per funzioni convesse sono strumenti utili nei problemi di ottimizzazione. Molti metodi di ottimizzazione godono di proprietà di convergenza solo locali; l’esistenza di un solo punto di minimo garantisce che minimi locali e globali coincidano (uno in questo caso), per cui anche metodi aventi proprietà di convergenza soltanto locale ereditano la convergenza globale dalle proprietà specifiche delle funzioni strettamente convesse.    


  1. Poiché f è continua in (a,b) per il teorema 3.4, per applicare il teorema di Weierstrass basta verificare che f sia continua in a e in b.

 
 

Punti di flesso

Leggi...

Definiamo ora il concetto di punto di flesso, cioè un punto in cui la funzione cambia la propria convessità o concavità. Nel seguito, per abuso di notazione, diremo che f'(x_0) = + \infty se

    \[\lim\limits_{x \to x_0^+} \frac{f(x) - f(x_0)}{x - x_0} = \lim\limits_{x \to x_0^-} \frac{f(x) - f(x_0)}{x - x_0} = + \infty.\]

Useremo analoga notazione per f'(x_0) = - \infty.

Definizione 5.1 (punti di flesso). Siano A \subseteq \mathbb{R}, f\colon A \to \mathbb{R} e x_0 \in A^{\circ}. Il punto x_0 si dice punto di flesso se esiste un intorno destro di x_0 in cui f è convessa (risp. concava) ed un intorno sinistro di x_0 in cui f è concava (risp. convessa).

    \[\quad\]

Osservazione 5.2. Geometricamente, se x_0 è un punto di flesso per la funzione f e la derivata f'(x_0) esiste, la tangente a f nel punto x_0 “attraversa” il grafico di f. In figura 9 è mostrato questo comportamento.

    \[\quad\]

    \[\quad\]

Figura 9: la tangente al grafico attraversa il grafico stesso in corrispondenza del punto di flesso A.

    \[\quad\]

    \[\quad\]

Osservazione 5.3. Dal teorema 2.10 segue direttamente che se f è derivabile due volte e x_0 è un punto di flessi per f, allora f''(x_0) = 0 e cambia segno in un intorno destro e sinistro di x_0. Infatti se f'' è continua, poiché f'' assume segni diversi da un lato e dall’altro di x_0, deve aversi f''(x_0)=0. Se invece f è soltanto derivabile due volte, il fatto che f''(x_0)= 0 segue dal teorema di Darboux [8, teorema 1.1]; supponiamo per fissare le idee che f sia convessa in un intorno sinistro di x_0 e concava in un intorno destro, allora f''(x)\geq 0 a sinistra e f''(x)\leq 0 a destra. Fissiamo x_1 nell’intorno destro di x_0 in cui f è concava. Per il teorema di Darboux, f'' assume tutti i valori compresi tra f''(x_0) e f''(x_1) nell’intervallo (x_0,x_1); poiché in tale intervallo f''(x) \leq 0, deve aversi f''(x_0) \leq 0. Analogamente si mostra che f''(x_0) \geq 0, da cui f''(x_0)=0.

I punti di flesso in cui esiste la derivata f' di f possono essere classificati in base al valore di tale derivata.

Definizione 5.4 (flesso orizzontale, verticale, obliquo). Sia A\subseteq \mathbb{R} e sia f\colon A \to \mathbb{R} una funzione. Sia x_0 un punto di flesso per f tale che esista f'(x_0) (finito o infinito). Allora x_0 si dice:

    \[\quad\]

  • punto di flesso orizzontale se la tangente a f in x_0 è orizzontale, ovvero se f'(x_0) = 0;
  •  

  • punto di flesso verticale se la tangente a f in x_0 è verticale, ovvero se f'(x_0) = \pm \infty;
  •  

  • punto di flesso obliquo se la tangente a f in x_0 è obliqua, ovvero se f'(x_0) \in \mathbb{R} \setminus \{0\}.

    \[\quad\]

    \[\quad\]

Figura 10: una funzione con un punto di flesso orizzontale (sinistra) e con un punto di flesso verticale (destra).

    \[\quad\]

    \[\quad\]

Osservazione 5.5. In figura 11 è mostrato un punto di flesso in cui la derivata non esiste. La funzione considerata è f\colon \mathbb{R} \to \mathbb{R} definita da

    \[f(x) = 	\begin{cases} 		\arctan(\frac{x}{2}), & x \geq 0 \\ 		\arctan(2x), & x < 0 	\end{cases}.\]

La funzione è chiaramente convessa nell’intervallo (-\infty, 0] e concava nell’intervallo [0, +\infty), dunque 0 è un punto di flesso per f, poiché la derivata di f non esiste in tale punto, esso non ricade in alcuna delle tipologie elencate dalla definizione 5.4.

    \[\quad\]

    \[\quad\]

Figura 11: La funzione in figura non è derivabile in 0, ma la concavità cambia tra l’intorno sinistro e destro di 0. Per cui 0 è un punto di flesso in cui la derivata non esiste.

    \[\quad\]


 
 

Approfondimenti

Introduzione.

In questa sezione analizzeremo due possibili approfondimenti sull’argomento delle funzioni convesse. Il primo riguarda il metodo di Newton, il secondo la disuguaglianza di Jensen.

Il metodo di Newton per funzioni convesse.

Esaminiamo ora il metodo di Newton, o metodo delle tangenti. Il metodo di Newton consente di calcolare le radici di un’equazione f(x) = 0, sotto opportune ipotesi su f\colon [a,b] \to \mathbb{R}. Descriviamo adesso intuitivamente il metodo di Newton, che è espresso formalmente nel teorema 6.1 nel quale si richiede che f sia continua, derivabile, convessa e che possegga uno zero in (a,b). Fissiamo il punto x_1 = b e tracciamo la retta r_1 tangente al grafico della funzione nel punto x_1. Tale retta ha equazione

    \[y = f(x_1) + f'(x_1)(x - x_1).\]

Se f'(x_1) \neq 0, chiamiamo x_2 il punto di intersezione di r_1 con l’asse delle ascisse, ovvero

    \[x_2 = x_1 - \frac{f(x_1)}{f'(x_1)}.\]

Sotto le ipotesi del teorema 6.1, si può mostrare che x_2 sia più vicino allo zero di f. Iterando il procedimento si definisce una successione in cui è stato posto x_1= b e

    \[x_{n+1} = x_n - \frac{f(x_n)}{f'(x_n)}.\]

Nel teorema 6.1 mostreremo che tale successione converge allo zero di f. In figura 12 è mostrato il procedimento che definisce i punti della successione \left\{x_n\right\}_{n \in \mathbb{N}}.

    \[\quad\]

    \[\quad\]

Figura 12: L’iterazione di Newton per determinare una successione convergente allo zero della funzione f. Partendo da x_1, tracciamo la tangente a f nel punto f(x_1) e chiamiamo x_2 l’intersezione di tale retta con l’asse delle ascisse (sinistra). Il nuovo punto x_2 è più vicino allo zero della funzione di x_1. Ripetiamo lo stesso procedimento per trovare x_3 (destra).

    \[\quad\]

Teorema 6.1 (metodo di Newton). Sia f\colon [a,b] \to \mathbb{R} una funzione strettamente crescente, convessa, continua in [a,b] e derivabile in (a,b]; supponiamo inoltre che f(a) < 0 < f(b).

Allora la successione \left\{x_n\right\} definita secondo l’iterazione di Newton

    \[\begin{cases} 					x_1 = b, \\ 					x_{n+1} = x_n - \dfrac{f(x_n)}{f'(x_n)}, 				\end{cases} \qquad \forall n \in \mathbb{N},\]

converge decrescendo all’unico zero di f(x) in [a,b].

    \[\quad\]

Dimostrazione. Osserviamo innanzitutto che, in virtù del teorema di esistenza degli zeri [9,teorema 1], lo zero x^* \in (a,b) esiste poiché f è continua e f(a)<0<f(b). Inoltre tale zero è unico per la monotonia stretta di f.

Proviamo prima che la successione x_n è ben definita in [a,b], che è decrescente, e poi mostriamo che essa converge a x^*.

    \[\quad\]

  • f'(x)>0 per ogni x \in (a,b). Sia x \in (a,b). Poiché f è strettamente crescente, applicando il teorema di Lagrange all’intervallo [a,x] si ottiene che esiste un punto \xi \in (a,x) tale che f'(\xi)>0. Poiché f è convessa, f' è crescente per il punto 3 del teorema 2.9 e dunque f'(x)> f'(\xi)>0.
  •  

  • x_n è ben definita in (a,b], decrescente e tale che f(x_n) \geq 0. Mostriamo per induzione tali proprietà.

    Ovviamente si ha x_1=b \in (a,b] e f(x_1)=f(b)>0 per ipotesi.

    Supponiamo ora che x_n \in (a,b] e che f(x_n) \geq 0. Dal punto precedente segue che f'(x_n)>0 e dunque l’espressione

    (61)   \begin{equation*} 			x_{n+1} = 			x_n - \frac{f(x_n)}{f'(x_n)} 		\end{equation*}

    che definisce x_{n+1} ha senso. Inoltre, da f(x_n) \geq 0 e da f'(x_n)>0, segue che x_{n+1} \leq x_n \leq b.

    Per provare che x_{n+1} \in (a,b], definiamo ora la funzione affine r \colon \mathbb{R} \to \mathbb{R} che descrive la retta tangente a f in x_n, ossia descritta da

    (62)   \begin{equation*} 			r(x) = f(x_n) + f'(x_n)(x-x_n) 			\qquad 			\forall x \in \mathbb{R}. 		\end{equation*}

    Si ha

    (63)   \begin{equation*} 			r(a) 			\leq 			f(a) 			< 			0 			= 			f(x_n) + f'(x_n) \left (x_n - \frac{f(x_n)}{f'(x_n)} - x_n \right ) 			= 			r(x_{n+1}) 			\leq 			f(x_{n+1}), 		\end{equation*}

    dove la prima e l’ultima disuguaglianza seguono dalla convessità di f per il punto 2 del teorema 2.9. Poiché r è una funzione crescente (il suo coefficiente angolare è f'(x_n)>0), da (63) si ottiene a< x_{n+1} e dunque x_{n+1} \in [a,b]. Inoltre, sempre da (63) segue che f(x_{n+1}) \geq 0.

  •  

  • x_n converge a x^*. Dalla monotonia di f e dal fatto che f(x_n) \geq 0 per ogni n \in \mathbb{N}, si ha

    (64)   \begin{equation*} 			x_n \geq x^* 			\qquad 			\forall n \in \mathbb{N}. 		\end{equation*}

    Poiché x_n è decrescente e limitata dal basso da x^*, essa è convergente per [1, teorema 6.1] e indichiamo con \bar{x} il suo limite. Sappiamo quindi che

    (65)   \begin{equation*} 			a < x^* \leq \bar{x} < b, 		\end{equation*}

    e vogliamo mostrare che \bar{x}= x^*. Riarrangiando l’espressione (61) si ottiene

    (66)   \begin{equation*} 			f'(x_n)(x_{n-1}-x_n) = - f(x_n) 			\qquad 			\forall n \in \mathbb{N}. 		\end{equation*}

    Passando al limite per n \to + \infty in questa equazione si ottiene

    (67)   \begin{equation*} 			0= -f(\bar{x}). 		\end{equation*}

    Infatti, il limite al membro di destra segue dalla continuità di f e per la caratterizzazione della continuità per successioni data da [3, teorema 3.3]. Invece il limite al membro di sinistra segue dal fatto che la successione f'(x_n) è limitata per la locale lipschitzianità di f in un intorno di \bar{x} data dal teorema 3.4, mentre

    (68)   \begin{equation*} 			\lim_{n \to + \infty} (x_{n+1}-x_n) 			= 			\bar{x} - \bar{x} 			= 			0, 		\end{equation*}

    da cui

    (69)   \begin{equation*} 			\lim_{n \to + \infty} f'(x_n)(x_{n+1}-x_n) 			= 			0. 		\end{equation*}

    Da (67) e dall’unicità dello zero x^* di f, segue che \bar{x}=x^*.

Il teorema 6.1 garantisce la convergenza della successione, sotto opportune ipotesi. Ma possiamo dire molto di più, ovvero stimare la velocità di convergenza del metodo di Newton rispetto al numero di iterazioni. Più precisamente, il prossimo risultato riguarda la velocità di convergenza del metodo di Newton.

Proposizione 6.2. Sia f\colon [a,b] \to \mathbb{R} una funzione strettamente crescente, convessa, derivabile due volte con derivata seconda continua. Supponiamo che f(a) < 0 < f(b), che m\coloneqq f'(a)>0 e M \coloneqq \max_{x \in [a, b]} f'(x). Allora, se x_n è la successione del teorema 6.1 e se \bar{x} è l’unico zero di f vale che

    \[x_n - \bar{x} \leq \frac{2m}{M}\left[\frac{M}{2m}(x_0 - \bar{x})\right]^{2^n} \qquad \forall n \in \mathbb{N}.\]

    \[\quad\]

Dimostrazione. Per costruzione, abbiamo che

(70)   \begin{equation*} 		f(x_n) = f'(x_n) (x_n - x_{n+1}). 	\end{equation*}

Per la formula di Taylor con resto di Lagrange, per ogni n \in \mathbb{N} esiste z_n \in (\bar{x}, x_n) tale che

(71)   \begin{equation*} 		0 = f(\bar{x}) = f(x_n) + f'(x_n)(\bar{x} - x_n) + \frac{f''(z_n)}{2}(\bar{x} - x_n)^2. 	\end{equation*}

Mettendo insieme le equazioni (70) e (71) otteniamo:

    \[0 = f'(x_n)(x_n - x_{n+1}) + f'(x_n)(\bar{x} - x_n) + \frac{f''(z_n)}{2}(\bar{x} - x_n)^2,\]

da cui segue che

    \[x_{n+1} - \bar{x} = \frac{f''(z_n)}{2f'(x_n)}(\bar{x} - x_n)^2.\]

Definendo \varepsilon_n = x_n - \bar{x} \geq 0 l’errore al passo n, per ogni n \in \mathbb{N}, abbiamo che

    \[0 \leq \varepsilon_{n+1} = \frac{f''(z_n)}{2f'(x_n)}\varepsilon_n^2 \leq \frac{M}{2m}\varepsilon_n^2.\]

A questo punto, per concludere la dimostrazione, usiamo un ragionamento per induzione. Per n = 0 la tesi diventa semplicemente x_0 - \bar{x} \leq x_0 - \bar{x} che è banalmente vera. Assumendo l’ipotesi induttiva, si ha che

    \[\varepsilon_{n+1} \leq \frac{M}{2m}\varepsilon_n^2 \leq \frac{2m}{M}\left[\frac{M}{2m}(x_0 - \bar{x})\right]^{2^{n+1}}\]

che è esattamente la tesi.

Osservazione 6.3. L’ipotesi f'(a)>0 non è restrittiva in quanto per la monotonia e la convessità di f, esiste un valore a' \in (a,\bar{x}) tale che f(a')>0. Dunque, a meno di restringere leggermente l’invervallo di partenza, si può assumere che m=f'(a)>0, in modo che la stima abbia senso.

Osservazione 6.4. L’ipotesi che f sia derivabile due volte nella proposizione 6.2 può essere sostituita con la derivabilità di f una volta e l’ipotesi che f' sia Lipschitziana: in tal caso la costante M si può sostituire con la costante di Lipschitz di f'; infatti

    \[0=f(x_n) + f'(\xi)(\bar{x}-x_n)\]

e si scrive

    \[f'(x_n) \leq f'(\xi) + M(x_n - \xi) 	\leq f'(\xi) + M(x_n - \bar{x}),\]

da cui si ottiene

    \[0 \leq f(x_n) + f'(x_n)(\bar{x}- x_n) + M(\bar{x}- x_n) ^2.\]

Ragionando come prima, si giunge alla medesima conclusione.

Osservazione 6.5. Il membro di destra nella stima della proposizione 6.2 converge a zero (e quindi fornisce una stima utile per l’errore) se e solo se il punto iniziale x_0 è scelto in modo che la quantità tra parentesi quadre sia minore di 1. Poiché x_n \to \bar x, sicuramente esiste N \in \mathbb{N} tale che tale disuguaglianza è valida per x_N. Considerando tale punto come punto iniziale, la stima diventa significativa.

Nel seguito mostriamo due esempi di applicazione del metodo di Newton.

Esempio 6.6. Si consideri l’equazione

    \[x^3 + 3x -5 = 0.\]

Vogliamo applicare il metodo di Newton alla funzione f \colon [0,2]\to \mathbb{R} definita da

    \[f(x) \coloneqq x^3 + 3x -5 \qquad \forall x\in [0,2].\]

Osserviamo che f possiede uno zero poiché f(0)= -5 e f(2) = 9, dunque f(0)\cdot f(2)<0. Si noti che f soddisfa le ipotesi del teorema 6.1, infatti f è derivabile due volte con derivata seconda continua (in quanto f è una funzione polinomiale), f è crescente poiché

    \[f'(x) = 3x^2 +3 > 0 \qquad \forall x \in [0,2], \]

inoltre f è convessa e f'(0)= 3 >0. Nel seguito costruiremo la successione del teorema 6.1 partendo non dal punto b ma dal punto a; il procedimento è analogo.

Dato x_0 = 0, abbiamo che f(x_0) = -5 e f'(x_0) = 3. Per cui:

    \[ x_1 = x_0 - \frac{f(x_0)}{f'(x_0)} =0 - \frac{-5}{3} = \frac{5}{3} \approx 1.67. \]

Ancora,

    \[f(x_1) = \frac{125}{27} \qquad \text{e} \qquad f'(x_1) = \frac{25}{3} +3 = \frac{34}{3}.\]

Perciò:

    \[ x_2 = \frac{5}{3} - \frac{125}{27}\frac{3}{34} = \frac{385}{306} \approx 1.26. \]

Dunque f(x_2) \approx 0.766, f'(x_2) \approx 7.749 e sostituendo come sopra otteniamo che x_3 \approx 1.16, mentre l’approssimazione alla terza cifra decimale della soluzione analitica è 1.154.

Esempio 6.7. Si consideri l’equazione

    \[x^5 + 5x -1 = 0.\]

Vogliamo applicare il metodo di Newton alla funzione f \colon [0,1]\to \mathbb{R} definita da

    \[f(x) \coloneqq x^5 + 5x -1 \qquad \forall x\in [0,1].\]

Osserviamo che f(0) = -1 e f(1) = 5, dunque f(0)\cdot f(1)<0, da cui segue che f ammette uno zero. Si noti che f soddisfa le ipotesi del teorema 6.1, infatti f è derivabile due volte con derivata seconda continua (in quanto f è una funzione polinomiale), f è crescente poiché

    \[f'(x) = 5x^4 + 5 > 0 \qquad \forall x \in [0,1], 	\]

inoltre f è convessa e f'(0)= 5 >0. Anche in questo esempio costruiremo la successione del teorema 6.1 partendo non dal punto b ma dal punto a; il procedimento è analogo.

Sia x_0 = 0, per cui f(x_0) = -1, f'(x_0) = 5, quindi

    \[x_1 = \frac{1}{5}, \qquad  f(x_1) \approx 3\cdot 10^{-4} \quad \text{e}\quad  f'(x_1) = 5.008.\]

Ne segue che x_2 \approx 0.199, che è esattamente l’approssimazione alla terza cifra decimale della soluzione analitica.


Disuguaglianza di Jensen.

Definizione 6.8 (media pesata). La media pesata di x_1,\dots, x_n \in \mathbb{R} e pesi \alpha_1, \dots, \alpha_n \in (0,1) tali che \alpha_1+\dots+\alpha_n = 1 è la somma

    \[\sum_{i=1}^n \alpha_i x_i.\]

    \[\quad\]

Osservazione 6.9. Nel caso in cui \alpha_1 = \dots = \alpha_n = \dfrac{1}{n} la media pesata coincide con la classica media aritmetica.

La disuguaglianza di convessità si può scrivere anche per più di 2 punti. In tal caso, essa prende il nome di disuguaglianza di Jensen.

Teorema 6.10 (disuguaglianza di Jensen). Sia I\subseteq \mathbb{R} un intervallo e sia f\colon I \to \mathbb{R} una funzione convessa; allora per ogni n \in \mathbb{N}, per ogni x_1, \dots, x_n \in I e per \alpha_1,\dots, \alpha_n \in[0,1] tale che \sum_{i=1}^n \alpha_i = 1, si ha

    \[f(\alpha_1 x_1 + \dots + \alpha_n x_n) \leq \alpha_1 f(x_1) + \dots + \alpha_n f(x_n).\]

    \[\quad\]

Dimostrazione. Si procede per induzione sul numero n di punti. Per n=2 la disuguaglianza si riconduce alla definizione 1.1 di convessità. Supponiamo quindi dimostrata la disuguaglianza per n-1 punti e mostriamo che essa è valida per n punti. Siano x_1, \dots, x_n \in I e siano \alpha_1, \dots, \alpha_n\in[0,1] tali che

    \[ 	\sum_{i=1}^n \alpha_i = 1. 	\]

Senza perdita di generalità, si può supporre che \alpha_1 \neq 1, altrimenti ci si ridurrebbe al caso di un unico punto. Quindi si ha

(72)   \begin{equation*} 		f\left(\alpha_1x_1 + \dots + \alpha_nx_n\right) = f\left(\alpha_1 x_1 + (1-\alpha_1) \sum_{i=2}^n\frac{\alpha_i}{1 - \alpha_1}x_i\right). 	\end{equation*}

Poiché

    \[\displaystyle \sum_{i=2}^n\frac{\alpha_i}{1 - \alpha_1} = 1\]

e I è un intervallo, il punto

    \[\displaystyle \xi= \sum_{i=2}^n\frac{\alpha_i}{1 - \alpha_1}x_i\]

è una combinazione convessa di punti di I, quindi \xi \in I. Infatti, se a, b \in I sono tali che a \leq x_2, \dots, x_n \leq b, si ha

(73)   \begin{equation*} \begin{split} 		a & = \sum_{i=2}^n\frac{\alpha_i}{1-\alpha_1}a \\ 		& \leq \sum_{i=2}^n\frac{\alpha_i}{1-\alpha_1}x_i \left( = \xi\right) \\ 		& \leq \sum_{i=2}^n\frac{\alpha_i}{1-\alpha_1}b \\ 		& = b. 	\end{split} 	\end{equation*}

Perciò \xi \in [a, b] \subseteq I. Dall’equazione (72) si ottiene

(74)   \begin{equation*} \begin{split} 		f\left(\alpha_1x_1 + \dots + \alpha_nx_n\right) & \leq \alpha_1f(x_1) + (1-\alpha_1)f(\xi) \\ 		& \leq \alpha_1f(x_1) + (1-\alpha_1)\sum_{i=2}^n\frac{\alpha_i}{1-\alpha_1}f(x_i) \\ 		& = \sum_{i=1}^n\alpha_if(x_i), \end{split} 	\end{equation*}

dove nella prima disuguaglianza abbiamo usato la definizione di convessità applicata ai punti x_1, \xi dell’equazione (72), mentre nella seconda disuguaglianza abbiamo applicato l’ipotesi induttiva.

Dal teorema 6.10 si ottengono direttamente i seguenti corollari, con opportune scelte dei coefficienti.

Corollario 6.11. Sia I\subseteq \mathbb{R} un intervallo e sia f\colon I \to \mathbb{R} una funzione convessa, allora per ogni x_1, \dots, x_n \in I e per ogni \alpha_1, \dots, \alpha_n > 0 si ha

    \[f\left(\frac{\alpha_1 x_1 + \dots + \alpha_n x_n}{\alpha_1 + \dots + \alpha_n}\right) \leq \frac{\alpha_1 f(x_1) + \dots + \alpha_n f(x_n)}				{\alpha_1 + \dots \alpha_n}\]

    \[\quad\]

Dimostrazione. Siccome \alpha_1, \dots \alpha_n > 0, ponendo

    \[\displaystyle \beta_i := \frac{\alpha_i}{\alpha_1+ \dots + \alpha_n}\]

per ogni i = 1, \dots, n abbiamo che \beta_1 + \dots + \beta_n = 1 e la tesi segue direttamente dal teorema 6.10.

Corollario 6.12. Sia I\subseteq \mathbb{R} un intervallo e sia f\colon I \to \mathbb{R} una funzione convessa, allora per ogni x_1,\dots, x_n\in I si ha

    \[f\left(\frac{x_1 + \dots + x_n}{n} \right) \leq \frac{1}{n}\left(f(x_1) + \dots + f(x_n) \right).\]

    \[\quad\]

Dimostrazione. Si applica il teorema 6.10 con \alpha_1 = \dots = \alpha_n = \dfrac{1}{n}.

 

Scarica la teoria

Ottieni la dispensa teorica sulle funzioni convesse di 18 pagine.

 
 

Esercizi

Esercizio 7.1  (\bigstar\bigstar\largewhitestar\largewhitestar\largewhitestar). Sia f\colon[0, +\infty) \to \mathbb{R} una funzione monotona crescente. Si provi allora che la funzione g\colon[0, +\infty) \to \mathbb{R} definita da

    \[ 		g(x) \coloneqq \int_{0}^{x}f(t)\,dt \qquad \forall x \in [0, +\infty) 		\]

è una funzione convessa.

Svolgimento.

La funzione g è ben definita, in quanto le funzioni monotone sono integrabili secondo Riemann, si veda ad esempio [1, teorema 5.1]. Consideriamo quindi a,b \in [0,+\infty) con a \leq b e \alpha \in [0,1]; vogliamo dimostrare che

(75)   \begin{equation*} 		g \big(\alpha a + (1-\alpha)b \big) 		\leq 		\alpha g(a) + (1-\alpha)g(b). 	\end{equation*}

Si ha

(76)   \begin{equation*} 		\begin{split} 			g \big(\alpha a + (1-\alpha)b \big) - g(a) 			= & 			\int_0^{\alpha a + (1-\alpha)b } f(t) \,\mathrm{d}t 			- 			\int_0^{a} f(t) \,\mathrm{d}t 			\\ 			= & 			\int_a^{\alpha a + (1-\alpha)b } f(t) \,\mathrm{d}t 			\\ 			\leq & 			\big(\alpha a + (1-\alpha)b - a \big) f\big( \alpha a + (1-\alpha)b \big) 			\\ 			= & 			(1-\alpha)(b-a) f\big( \alpha a + (1-\alpha)b \big). 		\end{split} 	\end{equation*}

dove la disuguaglianza segue dal fatto che f è monotona crescente in [a, \alpha a + (1-\alpha)b ] e dal fatto che lunghezza dell’intervallo di integrazione è pari a \alpha a + (1-\alpha)b - a. In maniera esattamente analoga si prova che

(77)   \begin{equation*} 		g(b) - g \big(\alpha a + (1-\alpha)b \big) \geq 		\alpha (b-a) f\big( \alpha a + (1-\alpha)b \big). 	\end{equation*}

Moltiplicando la disuguaglianza (76) per \alpha, moltiplicando la disuguaglianza (77) per 1-\alpha e confrontandole tra di loro si ottiene

(78)   \begin{equation*} 		\alpha g \big(\alpha a + (1-\alpha)b \big) - \alpha g(a) 		\leq 		(1-\alpha) g(b) - (1-\alpha)g \big(\alpha a + (1-\alpha)b \big). 	\end{equation*}

Riarrangiando i termini si ottiene (75), cioè la tesi.


Esercizio 7.2  (\bigstar\largewhitestar\largewhitestar\largewhitestar\largewhitestar). Sia a \in (0,+\infty) e sia f\colon \mathbb{R} \to \mathbb{R} una funzione definita da

    \[f(x) \coloneqq a^x \qquad \forall x \in \mathbb{R}.\]

Studiare la convessità di f.

Svolgimento.

Poiché f è derivabile due volte, possiamo applicare il teorema 2.10 per verificare la convessità di f studiando il segno della derivata seconda. Abbiamo che:

(79)   \begin{equation*} \begin{split} 	f'(x) = \log(a)a^x & \qquad \forall x \in \mathbb{R},\\ 	f''(x) = \log^2(a) a^x& \qquad \forall x \in \mathbb{R}, \end{split} \end{equation*}

da cui

    \[f''(x) >0  \quad \forall x \in \mathbb{R} \Longleftrightarrow a\in (0,1)\cup (1,+\infty),\]

e

    \[f''(x) = 0 \quad \forall x \in \mathbb{R} \Longleftrightarrow a = 1.\]

Perciò per il teorema 2.10 f è convessa per ogni a \in (0,+\infty) ed è strettamente convessa se a\in (0,1)\cup (1,+\infty).


Esercizio 7.3  (\bigstar\largewhitestar\largewhitestar\largewhitestar\largewhitestar). Sia f\colon \mathbb{R} \to \mathbb{R} definita da

    \[f(x) \coloneqq \sin(x) \qquad \forall x\in \mathbb{R}.\]

Studiare la convessità di f.

Svolgimento.

La funzione f è derivabile due volte, per cui possiamo applicare il teorema 2.10 e studiare il segno di f''. Si ha che

(80)   \begin{equation*} \begin{split} 	f'(x) = \cos(x) &\qquad \forall x \in \mathbb{R}\\ 	f''(x) =-\sin(x) &\qquad \forall x \in \mathbb{R}, \end{split} \end{equation*}

da cui

    \[f''(x) \ge 0 \Longleftrightarrow \sin(x) \leq 0 \Longleftrightarrow x \in [(2k - 1)\pi, 2k\pi], k \in \mathbb{Z}. \]

Segue quindi che f non è convessa in tutto \mathbb{R}, ma solo negli intervalli [(2k - 1)\pi, 2k\pi], k \in \mathbb{Z}.


Esercizio 7.4  (\bigstar\bigstar\largewhitestar\largewhitestar\largewhitestar). Sia a\in (0,1)\cup (1,+\infty) e sia f\colon (0, +\infty) \to \mathbb{R} definita da

    \[f(x) \coloneqq \log_a(x) \qquad \forall x\in (0, +\infty).\]

Studiare la convessità di f.

Svolgimento.

La funzione f è derivabile due volte, per cui possiamo applicare il teorema 2.10 e studiare il segno di f''. Si ha che

(81)   \begin{equation*} \begin{split} 	f'(x) = \frac{1}{x}\frac{1}{\log(a)} &\qquad \forall x \in \mathbb{R}, \\ 	f''(x) = - \frac{1}{x^2}\frac{1}{\log(a)} &\qquad \forall x \in \mathbb{R}, \end{split} \end{equation*}

Osserviamo che se a\in (0,1), allora

    \[f''(x) > 0 \qquad \forall x \in (0, +\infty),\]

mentre se a>1 allora f''(x) < 0 per ogni x\in (0,+\infty). Segue dunque che f se a \in (0, 1) allora f è strettamente convessa, mentre se a>1 è strettamente concava.


Esercizio 7.5  (\bigstar\largewhitestar\largewhitestar\largewhitestar\largewhitestar). Sia f \colon \mathbb{R} \to \mathbb{R} definita da

    \[f(x) \coloneqq \dfrac{x^2 +1}{e^x} \qquad \forall x \in \mathbb{R}.\]

Studiare la convessità di f.

Svolgimento.

La funzione f è derivabile due volte, per cui possiamo applicare il teorema 2.10 e studiare il segno di f''. Si ha che

(82)   \begin{equation*} \begin{split} 		f'(x) = -e^{-x}(x-1)^2  &\qquad \forall x \in \mathbb{R}, \\ 		f''(x) = e^{-x} (3 - 4x + x^2) &\qquad \forall x \in \mathbb{R}, \end{split} \end{equation*}

Poiché

    \[ 	e^{-x} > 0 \qquad \forall x \in \mathbb{R}, 	\]

dobbiamo soltanto studiare il segno di x^2-4x+3. Le radici di questo polinomio di secondo grado sono x=1 e x=3 ed esso assume segno positivo per valori esterni a tali radici. Segue che f è strettamente convessa negli intervalli (-\infty, 1] \cup [3, +\infty) ed è strettamente concava nell’intervallo [1, 3].


Esercizio 7.6  (\bigstar\bigstar\largewhitestar\largewhitestar\largewhitestar). Per quali valori del parametro a \in \mathbb{R} la funzione f\colon\mathbb{R}\to\mathbb{R} definita da

    \[f(x) = \log(1+x^2) - \frac{a}{2}x^2 \qquad \forall x \in \mathbb{R}\]

è convessa?

Svolgimento.

Siccome f è derivabile due volte guardiamo il segno della derivata seconda per studiare la convessità di f, come stabilito dal teorema 2.10. Si ha

(83)   \begin{equation*} \begin{split} 	f'(x) = \frac{2x}{x^2+1} - ax &\qquad \forall x \in \mathbb{R}\\ 	f''(x) = -a-\frac{2(x^2 -1)}{(1 + x^2)^2} &\qquad \forall x \in \mathbb{R}. \end{split} \end{equation*}

Poiché

    \[f''(x)\geq0 \Longleftrightarrow a \leq  -\frac{2(x^2 - 1)}{(x^2+1)^2},  \quad \forall  x \in \mathbb{R}\]

occorre capire per quali a \in \mathbb{R} tale condizione è valida. Per brevità di notazione, definiamo la funzione g\colon\mathbb{R} \to \mathbb{R} definita da

    \[g(x) \coloneqq -\dfrac{2(x^2-1)}{(x^2+1)^2},\]

e determiniamo \mathrm{inf}_\mathbb{R} g. Poiché

    \[\lim_{x\to+\infty}g(x) = \lim_{x\to-\infty}g(x) = 0,\]

e poiché g continua e g\left(\dfrac{1}{2}\right)<0, tale funzione ammette minimo per [4, proposizione 4.5]; inoltre tale minimo viene assunto in un punto critico poiché g è derivabile ovunque. Calcoliamo dunque la derivata prima di g:

    \[ g'(x) = \frac{4(x^3-3x)}{(x^2+1)^3} \qquad \forall x \in \mathbb{R}, \]

da cui segue che

    \[g'(x) = 0 \Longleftrightarrow x=\{0, \sqrt{3}, -\sqrt{3}\}.\]

e g(0) = 0, g(\sqrt{3}) = g(-\sqrt{3}) = -\dfrac{1}{4}. Segue dunque che

    \[\min_\mathbb{R} g = -\frac{1}{4},\]

da cui se a\leq-\dfrac{1}{4} si ha che f''(x) \geq 0 per ogni x \in \mathbbR}. Quindi, per il teorema 2.10 f è convessa per tali valori di a.


Esercizio 7.7  (\bigstar\bigstar\largewhitestar\largewhitestar\largewhitestar). Sia f: \mathbb{R}\to (0,+\infty). Si dica quali delle seguenti affermazioni sono corrette:

    \[\quad\]

  1. f convessa implica che \log(f) è convessa;
  2.  

  3. \log(f) convessa implica che f è convessa.

Svolgimento.

  • Verifichiamo che l’affermazione non è corretta. Un controesempio immediato è considerare f\colon \mathbb{R} \to \mathbb{R} definita da f(x) \coloneqq x per ogni x\in \mathbb{R}. Dunque, è immediato vedere che f è convessa, poiché affine, ma \log x è strettamente concava.
  •  

  • L’affermazione è corretta. Infatti, è sufficiente notare che, poiché f è a valori positivi per ipotesi, possiamo riscrivere f = e^{\log f}. Dato che la funzione esponenziale \psi(y) = e^y è convessa e monotona crescente, e che \varphi = \log(f) è convessa per ipotesi, si conclude che f = \psi \circ \varphi è convessa per la proposizione 1.7.

Esercizio 7.8  (\bigstar\bigstar\largewhitestar\largewhitestar\largewhitestar). Si mostri che la funzione f\colon(0, +\infty) \to \mathbb{R} definita da f(x) \coloneqq -\log(x) per x \in (0,+\infty) è convessa e si usi tale fatto per dimostrare la seguente disuguaglianza, detta disuguaglianza di Young,

    \[ 				\left\vert x y \right\vert \leq  \dfrac{\left \vert x \right \vert ^p}{p}+\dfrac{\left \vert y \right \vert ^q}{q}\qquad\forall x,y \in \mathbb{R} 				\]

con p,\,q >1 tali che \dfrac{1}{p}+\dfrac{1}{q}=1.

Svolgimento.

La funzione f(x) = -\log(x) è derivabile due volte e la sua derivata seconda è f''(x) = 1/x^2, che è positiva per ogni x > 0. Di conseguenza, f è convessa per il teorema 2.10. Per definizione di convessità, quindi, presi x,y \in \mathbb{R} arbitrari:

    \[-\log\left(\frac{1}{p}|x|^p+\frac{1}{q}|y|^q\right) \leq -\frac{1}{p}\log\left(|x|^p\right) - \frac{1}{q}\log\left(|y|^q\right) = -\log\left(|xy|\right).\]

Moltiplicando per -1 ed esponenziando ambo i membri, per la monotonia del logaritmo, si ha

    \[\frac{1}{p}|x|^p+\frac{1}{q}|y|^q \geq |xy|,\]

che è la disuguaglianza di Young.


Esercizio 7.9  (\bigstar\largewhitestar\largewhitestar\largewhitestar\largewhitestar). Determinare i punti di flesso della funzione f\colon\mathbb{R} \to \mathbb{R} definita da

    \[f(x) = x e^x \qquad \forall x \in \mathbb{R}.\]

Svolgimento.

Abbiamo che:

    \[f'(x) = (x + 1)e^x \qquad \forall x \in \mathbb{R},\]

    \[f''(x) = (x + 2)e^x\qquad \forall x \in \mathbb{R}.\]

Per cui

    \[ f'' \geq 0 \qquad \forall x \in [-2, +\infty), \]

    \[ f'' \leq 0 \qquad \forall x \in (-\infty, -2]. \]

Quindi il punto x_0 = -2 è di flesso per f perché la derivata seconda cambia segno in un suo intorno ed è definita in x_0. Poiché la derivata prima esiste, è finita e non nulla in x_0, il punto è di flesso obliquo.


Esercizio 7.10  (\bigstar\bigstar\largewhitestar\largewhitestar\largewhitestar). Determinare i punti di flesso della funzione f\colon \mathbb{R} \to \mathbb{R} definita da

    \[f(x) = \sin(x) \qquad  \forall x \in \mathbb{R}.\]

Svolgimento.

Guardando le derivate abbiamo

    \[ f'(x) = \cos(x)\qquad \forall x \in \mathbb{R}, \]

    \[f''(x) = - \sin(x) \qquad \forall x \in \mathbb{R}. \]

La derivata seconda si annulla in tutti i punti del tipo x_k = k \pi, k \in \mathbb{Z} e cambia segno negli intorni destri e sinistri di tali punti, per cui sono effettivamente punti di flesso. Possiamo concludere che tali punti sono dei flessi obliqui in quanto gli zeri della derivata seconda non coincidono mai con quelli della derivata prima (escludendo quindi il caso di punto di flesso orizzontale) e il coseno è una funzione limitata (escludendo quindi il caso di punto di flesso verticale).


Esercizio 7.11  (\bigstar\bigstar\largewhitestar\largewhitestar\largewhitestar). Determinare i punti di flesso della funzione f\colon\mathbb{R} \to \mathbb{R} definita da

    \[f(x) = \frac{1}{1+x^2} \qquad \forall x \in \mathbb{R}.\]

Svolgimento.

Si ha

    \[ f'(x) = - \frac{2x}{(1+x^2)^2} \qquad \forall x \in \mathbb{R}, \]

    \[ f''(x) = - \frac{2}{(1+x^2)^2} + \frac{8x^2}{(1+x^2)^3} = \frac{2(3x^2 - 1)}{(1 + x^2)^3}  \qquad \forall x \in \mathbb{R}. \]

Perciò f''(x) = 0 nei punti x_1 = \dfrac{1}{\sqrt 3} e x_2 = -\dfrac{1}{\sqrt 3} e la derivata seconda cambia segno negli intorni destri e sinistri di tali punti. Vale che

    \[f'(x_1) = - \frac{3\sqrt{3}}{8}\qquad \text{e} \qquad f'(x_2) = \frac{3\sqrt{3}}{8}\]

e pertanto i due punti sono di flesso obliquo.


Esercizio 7.12  (\bigstar\largewhitestar\largewhitestar\largewhitestar\largewhitestar). È data la funzione f\colon\mathbb{R}\setminus\{0\}\to\mathbb{R} definita da

    \[f(x) = x^2 - 3x + \log|x| \qquad \forall x \in \mathbb\setminus\{0\}.\]

Determinare i punti di flesso di f.

Svolgimento.

Abbiamo che

    \[f'(x) = 2x + \frac{1}{x} - 3 \qquad \forall x \in \mathbb{R}\setminus\{0\}, \]

    \[ f''(x) = 2 - \frac{1}{x^2} \qquad \forall x \in \mathbb{R}\setminus\{0\}. \]

Per cui i possibili punti di flesso sono x_1 = \dfrac{1}{\sqrt 2} e x_2 = -\dfrac{1}{\sqrt 2}. Si ha che f'' cambia segno negli intorni dei suoi zeri, per cui sono entrambi punti di flesso. Siccome

    \[f'(x_1) = \frac{3}{\sqrt{2}} - 3 \qquad \text{e} \qquad f'(x_2) = -\frac{3}{\sqrt{2}} - 3,\]

i due punti sono di flesso obliquo.


Esercizio 7.13  (\bigstar\largewhitestar\largewhitestar\largewhitestar\largewhitestar). Determinare i punti di flesso della funzione f\colon\mathbb{R} \to \mathbb{R} definita da

    \[f(x) = \sqrt[3]{8 - x^3} \qquad \forall x \in \mathbb{R}.\]

Svolgimento.

La derivata prima di f risulta

    \[f'(x) = \frac{- x^2}{\sqrt[3]{(8 - x^3)^2}} \qquad \forall x \in \mathbb{R},\]

e la derivata seconda è

    \[f''(x) = \frac{- 16x}{(8 - x^3)\sqrt[3]{(8 - x^3)^2}} \qquad \forall x \in \mathbb{R}.\]

Abbiamo che f''(0) = 0, f'' > 0 in un intorno sinistro di 0 e f'' < 0 in un intorno destro di 0. Per cui 0 è un punto di flesso. Poiché f'(0) = 0, 0 è un punto di flesso a tangente orizzontale. Notiamo inoltre che f' e f'' non sono definite nel punto x = 2, ma lo sono in tutti i suoi intorni. Il segno di f'' è positivo in un intorno destro di 2 ed è negativo in un intorno sinistro di 2. Abbiamo che:

    \[f'(-2) = - \infty\]

per cui x = 2 è un punto di flesso verticale. La figura 13 mostra il grafico di f con i suoi due flessi.

    \[\quad\]

    \[\quad\]

Figura 13: il grafico della funzione f(x) = \sqrt[3]{8 - x^3}. Sono inoltre evidenziati i due punti di flesso e le relative rette tangenti.

    \[\quad\]


Esercizio 7.14  (\bigstar\bigstar\largewhitestar\largewhitestar\largewhitestar). Sia f \colon (a,b) \to \mathbb{R} una funzione convessa e derivabile. Allora f' è continua.

Svolgimento.

Fissiamo x_0 \in (a,b) e mostriamo che f' è continua in x_0. Per il teorema 2.9, sappiamo che f' è monotona crescente, quindi esistono i limiti

(84)   \begin{equation*} 		\ell_- 		\coloneqq 		\lim_{x \to x_0^-} f'(x) 		= 		\sup_{x < x_0} f'(x) 		\leq 		f'(x_0) 		\leq 		\inf_{x > x_0} f'(x) 		= 		\lim_{x \to x_0^+} f'(x) 		\eqqcolon 		\ell_+ 	\end{equation*}

Per il teorema di Darboux [8, teorema 1.1], la funzione derivata possiede la proprietà dei valori intermedi, quindi f' assume tutti i valori compresi nell’intervallo [\ell_-,\ell_+]. Per (84), ovvero per la monotonia di f', necessariamente allora tale intervallo deve essere costituito dal solo valore f'(x_0). Dunque

(85)   \begin{equation*} 		\lim_{x \to x_0^-} f'(x) 		= 		\ell_- 		= 		f'(x_0) 		= 		\ell_+ 		= 		\lim_{x \to x_0^+} f'(x), 	\end{equation*}

ossia f' è continua in x_0. Per l’arbitrarietà di x_0 \in (a,b), f' è continua.


Esercizio 7.15  (\bigstar\bigstar\bigstar\bigstar\largewhitestar). Diciamo che una funzione f \colon (a,b) \to \mathbb{R} è localmente convessa se, per ogni punto x \in (a,b), esiste \delta_x > 0 tale che f è convessa nell’intorno (x-\delta_x,x+\delta_x).

Mostrare che, se f è localmente convessa, allora è convessa.

Svolgimento.

Osserviamo innanzitutto che, dal teorema 3.4, e dall’ipotesi di locale convessità, per ogni punto x \in (a,b), f è continua in (x-\delta_x,x+\delta_x). Quindi f è continua in (a,b).

Siano x,y \in (a,b) con x < y e definiamo la funzione g \colon [x,y] \to \mathbb{R} come

(86)   \begin{equation*} 		g(t) 		= 		f(t) - f(x) + \frac{f(y)-f(x)}{y-x} (t-x) 		\qquad 		\forall t \in [x,y]. 	\end{equation*}

Per il punto 2 del teorema 2.3 e dall’arbitrarietà di x,y \in (a,b), è sufficiente mostrare che g(t) \leq 0 per ogni t \in [x,y].

Osserviamo innanzitutto che g è una funzione continua poiché somma di una funzione continua e di una funzione affine e che g è localmente convessa per la proposizione 1.7 poiché è somma di una funzione localmente convessa e una convessa. Inoltre vale g(x)=g(y)=0.

Supponiamo per assurdo che esista t \in [x,y] tale che g(t)>0. Per la continuità di g e per il teorema di Weierstrass, allora esiste M \coloneqq \max_{[x,y]}g >0. Definiamo dunque l’insieme

(87)   \begin{equation*} 		E 		= 		\{t \in [x,y] \colon g(t)=M\}, 	\end{equation*}

ossia l’insieme dei punti di massimo assoluto per g, e chiamiamo

(88)   \begin{equation*} 		\tau 		= 		\inf E. 	\end{equation*}

Per la continuità di g si ha che \tau deve soddisfare

(89)   \begin{equation*} 		g(\tau) = M, 	\end{equation*}

da cui segue in particolare che \tau \in (x,y) in quanto M>0, mentre g(x)=g(y)=0. Per la locale convessità di g, esiste un intorno (\tau - \delta_\tau, \tau + \delta_\tau)\subseteq (a,b) di \tau in cui g è convessa. Da (\tau - \delta_\tau, \tau + \delta_\tau)\subseteq (a,b) segue che \tau è un punto di massimo per g in (\tau - \delta_\tau, \tau + \delta_\tau), quindi per il principio del massimo dato dalla proposizione 4.6, g è costantemente pari a M in (\tau - \delta_\tau, \tau + \delta_\tau), ma ciò vuol dire che esiste t< \tau tale che g(t)=M, in contraddizione col fatto che \tau=\inf E.

Da tale contraddizione segue che g(t) \leq 0 per ogni t \in [x,y], come volevasi dimostrare.


Esercizio 7.16  (\bigstar\bigstar\largewhitestar\largewhitestar\largewhitestar). Sia I \subseteq \mathbb{R} un intervallo e siano f,g \colon I \to \mathbb{R} due funzioni convesse. Si provi che la funzione h \colon I \to \mathbb{R} definita da

(90)   \begin{equation*} 			h(x) 			= 			\max \{ f(x), g(x) \} 			\qquad 			\forall x \in I 		\end{equation*}

è convessa (si veda la figura 14).

Analogamente, se \mathcal{F} è un insieme di funzioni convesse aventi dominio I tali che la funzione h \colon I \to \mathbb{R} definita da

(91)   \begin{equation*} 			h(x) 			= 			\sup_{f \in \mathcal{F}} f(x) 			\qquad 			\forall x \in I 		\end{equation*}

sia finita ovunque, allora h è convessa.

Osservazione 7.17.

Notiamo che, nella seconda parte dell’esercizio 7.16, l’insieme \mathcal{F} può essere anche infinito e possedere qualunque cardinalità.

Svolgimento.

Siano x,y \in I e sia \alpha \in [0,1]. Si ha

(92)   \begin{equation*} 		\begin{gathered} 			f \big( \alpha x + (1-\alpha)y \big) 			\leq 			\alpha f(x) + (1-\alpha)f(y) 			\leq 			\alpha h(x) + (1-\alpha)h(y), 			\\ 			g \big( \alpha x + (1-\alpha)y \big) 			\leq 			\alpha g(x) + (1-\alpha)g(y) 			\leq 			\alpha h(x) + (1-\alpha)h(y). 		\end{gathered} 	\end{equation*}

dove le prime disuguaglianze sono dovute alla convessità di f e g, mentre le seconde disuguaglianze seguono da h= \max\{f,g\}. Da tali relazioni segue che

(93)   \begin{equation*} 		h \big( \alpha x + (1-\alpha)y \big) 		= 		\max \Big\{f \big( \alpha x + (1-\alpha)y \big), g \big( \alpha x + (1-\alpha)y \big)  \Big\} 		\leq 		\alpha h(x) + (1-\alpha)h(y), 	\end{equation*}

ossia, per l’arbitrarietà di x,y \in I e \alpha \in [0,1], che h è convessa.

    \[\quad\]

    \[\quad\]

Figura 14: illustrazione dell’esercizio 7.16; se f e g sono funzioni convesse allora la funzione massimo h=\max\{f,g\}, rappresentata in verde, è convessa.

    \[\quad\]

In maniera analoga, sia \mathcal{F} un insieme di funzioni convesse aventi dominio I e supponiamo che h=\sup_{f \in \mathcal{F}} f sia ovunque finita. Fissando nuovamente x,y \in I e \alpha \in [0,1], si ha

(94)   \begin{equation*} 		f \big( \alpha x + (1-\alpha)y \big) 		\leq 		\alpha f(x) + (1-\alpha)f(y) 		\leq 		\alpha h(x) + (1-\alpha)h(y) 		\qquad 		\forall f \in \mathcal{F}, 	\end{equation*}

dove la prima disuguaglianza deriva dalla convessità di f, mentre la seconda da h = \sup_{\mathcal{F}} f. Poiché l’ultimo membro di (94) non dipende da f, passando all’estremo superiore nel primo membro al variare di f \in \mathcal{F}, si ottiene

(95)   \begin{equation*} 		h \big( \alpha x + (1-\alpha)y \big) 		= 		\sup_{f \in \mathcal{F}}  \Big\{f \big( \alpha x + (1-\alpha)y \big)\Big\} 		\leq 		\alpha h(x) + (1-\alpha)h(y). 	\end{equation*}

Di nuovo ciò implica la convessità di h.


Esercizio 7.18  (\bigstar\bigstar\bigstar\largewhitestar\largewhitestar). Sia I \subseteq \mathbb{R} un intervallo e sia f \colon I \to \mathbb{R} una funzione. Si provi che le seguenti affermazioni sono equivalenti:

  1. f è convessa;
  2. f è estremo superiore di funzioni affini, ossia esiste un insieme \mathcal{F} costituito da funzioni affini e aventi dominio I tali che

    (96)   \begin{equation*} 				f(x) 				= 				\sup_{g \in \mathcal{F}} g(x) 				\qquad 				\forall x \in I. 			\end{equation*}

Svolgimento.

Mostriamo separatamente le due implicazioni.

    \[\quad\]

  • 1 \Rightarrow 2. Definiamo

    (97)   \begin{equation*} 			\mathcal{F} 			\coloneqq 			\left \{  g \colon I \to \mathbb{R}, \,\,\, g(x) \leq f(x) \,\,\forall x \in I,\,\,\, g \text{ affine} \right \}, 		\end{equation*}

    ovvero \mathcal{F} è l’insieme delle funzioni affini che sono ovunque minori o uguali a f. Definiamo inoltre la funzione h \colon I \to \mathbb{R} come l’estremo superiore delle funzioni \mathcal{F}, cioè

    (98)   \begin{equation*} 			h(x) 			= 			\sup_{g \in \mathcal{F}} g(x) 			\qquad 			\forall x \in I. 		\end{equation*}

    Poiché ogni funzione g \in \mathcal{F} è limitata dall’alto da f, si ha

    (99)   \begin{equation*} 			h(x) 			= 			\sup_{g \in \mathcal{F}} g(x) 			\leq f(x) 			\qquad 			\forall x \in I. 		\end{equation*}

    In particolare h è finita ovunque. Mostriamo ora che h\geq f, che prova l’asserto. A tal fine, fissiamo x_0 \in I^{\mathrm{o}} (il caso al bordo è analogo) e mostriamo che h(x_0)=f(x_0). Infatti, per il lemma 3.6 in x_0 esistono f'_-(x_0), f'_+(x_0), cioè le derivate sinistra e destra di f in x_0 e vale

    (100)   \begin{equation*} 			f'_-(x_0) \leq f'_+(x_0). 		\end{equation*}

    Fissiamo m tale che f'_-(x_0) \leq m \leq f'_+(x_0) e definiamo la funzione affine g \colon I \to \mathbb{R} come

    (101)   \begin{equation*} 			g(x) 			= f(x_0) + m (x- x_0) 			\qquad 			\forall x \in I. 		\end{equation*}

    Si ha ovviamente g(x_0)=f(x_0), dunque rimane da dimostrare che g(x) \leq f(x) per ogni x \in I. Cominciamo a osservare che

    (102)   \begin{gather*} 			f(x) 			= 			f(x_0) + \frac{f(x)-f(x_0)}{x-x_0} (x-x_0) 			\geq 			f(x_0) + f'_-(x_0) (x-x_0) 			\geq 			g(x) 			\qquad 			\forall x \in I,\,\, x < x_0,\qquad  			\\ \end{gather*}

    (103)   \begin{gather*} 			g(x) 			\leq 			f(x_0) + f'_-(x_0) (x-x_0) 			\leq 			f(x_0) + \frac{f(x)-f(x_0)}{x-x_0} (x-x_0) 			= 			f(x) 			\qquad 			\forall x \in I, \,\, x > x_0,\qquad  		\end{gather*}

    dove le disuguaglianze seguono dalla scelta di m e dalla monotonia dei rapporti incrementali data dal punto 4 del teorema 2.3, che implicano

    (104)   \begin{equation*} 			\begin{gathered} 				m \geq 				f'_-(x_0) 				= 				\lim_{x \to x_0^-} \frac{f(x)-f(x_0)}{x-x_0} 				\geq 				\frac{f(x)-f(x_0)}{x-x_0} 				\qquad 				\forall x \in I, \,\, x < x_0, 				\\ 				m \leq 				f'_+(x_0) 				= 				\lim_{x \to x_0^+} \frac{f(x)-f(x_0)}{x-x_0} 				\leq 				\frac{f(x)-f(x_0)}{x-x_0} 				\qquad 				\forall x \in I, \,\, x > x_0, 			\end{gathered} 		\end{equation*}

    Unendo (102) e (103), si ottiene

    (105)   \begin{equation*} 			f(x) 			\geq 			g(x) 			\qquad 			\forall y \in I, 		\end{equation*}

    ossia g \in \mathcal{F}. Da ciò e da (105) si ha che anche h(x_0) \geq f(x_0) e quindi h=f.

  •  

  • 2 \Rightarrow 1. Poichè tutte le funzioni affini sono convesse, f= \sup_{\mathcal{F}} g è convessa in quanto estremo superiore di funzioni convesse, per il secondo punto dell’esercizio 7.16.

 
 

Riferimenti bibliografici

[1] Giusti, E., Analisi matematica 1, Bollati Boringhieri (1988).

[2] Qui Si Risolve, Funzioni elementari — Volume 1.

[3] Qui Si Risolve, Funzioni continue.

[4] Qui Si Risolve, Funzioni continue – esercizi.

[5] Qui Si Risolve, Calcolo differenziale — Teoria sulle derivate.

[6] Qui Si Risolve, Limite di una successione monotona.

[7] Qui Si Risolve, Il teorema di Fermat.

[8] Qui Si Risolve, Il teorema di Darboux.

[9] Qui Si Risolve, Il teorema di esistenza degli zeri.

 
 

Tutta la teoria di analisi matematica

Leggi...

  1. Teoria Insiemi
  2. Il metodo della diagonale di Cantor
  3. Logica elementare
  4. Densità dei numeri razionali nei numeri reali
  5. Insiemi Numerici \left(\mathbb{N},\, \mathbb{Z},\, \mathbb{Q}\right)
  6. Il principio di induzione
  7. Gli assiomi di Peano
  8. L’insieme dei numeri reali: costruzione e applicazioni
  9. Concetti Fondamentali della Retta Reale: Sintesi Teorica
  10. Costruzioni alternative di \mathbb{R}
  11. Binomio di Newton
  12. Spazi metrici, un’introduzione
  13. Disuguaglianza di Bernoulli
  14. Disuguaglianza triangolare
  15. Teoria sulle funzioni
  16. Funzioni elementari: algebriche, esponenziali e logaritmiche
  17. Funzioni elementari: trigonometriche e iperboliche
  18. Funzioni goniometriche: la guida essenziale
  19. Teorema di Bolzano-Weierstrass per le successioni
  20. Criterio del rapporto per le successioni
  21. Definizione e proprietà del numero di Nepero
  22. Limite di una successione monotona
  23. Successioni di Cauchy
  24. Il teorema ponte
  25. Teoria sui limiti
  26. Simboli di Landau
  27. Funzioni continue – Teoria
  28. Il teorema di Weierstrass
  29. Il teorema dei valori intermedi
  30. Il teorema della permanenza del segno
  31. Il teorema di Heine-Cantor
  32. Il teorema di esistenza degli zeri
  33. Il metodo di bisezione
  34. Teorema ponte versione per le funzioni continue
  35. Discontinuità di funzioni monotone
  36. Continuità della funzione inversa
  37. Teorema delle contrazioni o Teorema di punto fisso di Banach-Caccioppoli
  38. Teoria sulle derivate
  39. Calcolo delle derivate: la guida pratica
  40. Teoria sulle funzioni convesse
  41. Il teorema di Darboux
  42. I teoremi di de l’Hôpital
  43. Teorema di Fermat
  44. Teoremi di Rolle e Lagrange
  45. Il teorema di Cauchy
  46. Espansione di Taylor: teoria, esempi e applicazioni pratiche
  47. Polinomi di Taylor nei limiti: istruzioni per l’uso
  48. Integrali definiti e indefiniti
  49. Teorema fondamentale del calcolo integrale (approfondimento)
  50. Integrali ricorsivi
  51. Formule del trapezio, rettangolo e Cavalieri-Simpson
  52. Teoria sugli integrali impropri
  53. Funzioni integrali – Teoria
  54. Introduzione ai numeri complessi – Volume 1 (per un corso di ingegneria — versione semplificata)
  55. Introduzione ai numeri complessi – Volume 1 (per un corso di matematica o fisica)
  56. Serie numeriche: la guida completa
  57. Successioni di funzioni – Teoria
  58. Teoremi sulle successioni di funzioni
    1. 58a. Criterio di Cauchy per la convergenza uniforme
    2. 58b. Limite uniforme di funzioni continue
    3. 58c. Passaggio al limite sotto il segno di integrale
    4. 58d. Limite uniforme di funzioni derivabili
    5. 58e. Piccolo teorema del Dini
    6. 58f. Procedura diagonale e teorema di Ascoli-Arzela
  59. Serie di funzioni – Teoria
  60. Serie di potenze – Teoria
  61. Serie di Fourier – Teoria e applicazioni
  62. Integrali multipli — Parte 1 (teoria)
  63. Integrali multipli — Parte 2 (teoria e esercizi misti)
  64. Regola della Catena — Teoria ed esempi.
  65. Jacobiano associato al cambiamento di coordinate sferiche
  66. Guida ai Massimi e Minimi: Tecniche e Teoria nelle Funzioni Multivariabili
  67. Operatore di Laplace o Laplaciano
  68. Teoria equazioni differenziali
  69. Equazione di Eulero
  70. Teoria ed esercizi sulla funzione Gamma di Eulero
  71. Teoria ed esercizi sulla funzione Beta
  72. Approfondimento numeri complessi
  73. Diverse formulazioni dell’assioma di completezza
  74. Numeri di Delannoy centrali
  75. Esercizi avanzati analisi

 
 

Tutte le cartelle di Analisi Matematica

Leggi...

  1. Prerequisiti di Analisi
    1. Ripasso algebra biennio liceo
    2. Ripasso geometria analitica
    3. Ripasso goniometria e trigonometria
    4. Errori tipici da evitare
    5. Insiemi numerici N,Z,Q,R
    6. Funzioni elementari
    7. Logica elementare
    8. Insiemi
  2. Successioni
    1. Teoria sulle Successioni
    2. Estremo superiore e inferiore
    3. Limiti base
    4. Forme indeterminate
    5. Limiti notevoli
    6. Esercizi misti Successioni
    7. Successioni per ricorrenza
  3. Funzioni
    1. Teoria sulle funzioni
    2. Verifica del limite in funzioni
    3. Limite base in funzioni
    4. Forme indeterminate in funzioni
    5. Limiti notevoli in funzioni
    6. Calcolo asintoti
    7. Studio di funzione senza derivate
    8. Dominio di una funzione
    9. Esercizi misti Funzioni
    10. Esercizi misti sui Limiti
  4. Funzioni continue-lipschitziane-holderiane
    1. Teoria sulle Funzioni continue-lipschitziane-holderiane
    2. Continuità delle funzioni
    3. Continuità uniforme
    4. Teorema degli zeri
    5. Esercizi sul teorema di Weierstrass senza l’uso delle derivate
  5. Calcolo differenziale
    1. Derivate
    2. Calcolo delle derivate
    3. Retta tangente nel calcolo differenziale
    4. Punti di non derivabilità nel calcolo differenziale
    5. Esercizi sul teorema di Weierstrass con l’uso delle derivate
    6. Studio di funzione completo nel calcolo differenziale
    7. Esercizi teorici nel calcolo differenziale
    8. Metodo di bisezione
    9. Metodo di Newton
  6. Teoremi del calcolo differenziale
    1. Teoria sui Teoremi del calcolo differenziale
    2. Teorema di Rolle
    3. Teorema di Lagrange
    4. Teorema di Cauchy
    5. Teorema di De L’Hôpital
  7. Calcolo integrale
    1. Integrale di Riemann
    2. Integrali immediati
    3. Integrale di funzione composta
    4. Integrali per sostituzione
    5. Integrali per parti
    6. Integrali di funzione razionale
    7. Calcolo delle aree
    8. Metodo dei rettangoli e dei trapezi
    9. Esercizi Misti Integrali Indefiniti
    10. Esercizi Misti Integrali Definiti
  8. Integrali impropri
    1. Teoria Integrali impropri
    2. Carattere di un integrale improprio
    3. Calcolo di un integrale improprio
  9. Espansione di Taylor
    1. Teoria Espansione di Taylor
    2. Limiti di funzione con Taylor
    3. Limiti di successione con Taylor
    4. Stime del resto
  10. Funzioni integrali (Approfondimento)
    1. Teoria Funzioni integrali (Approfondimento)
    2. Studio di funzione integrale
    3. Limiti con Taylor e De L’Hôpital
    4. Derivazione di integrali parametrici (Tecnica di Feynmann)
  11. Numeri Complessi
    1. Teoria Numeri complessi
    2. Espressioni con i numeri complessi
    3. Radice di un numero complesso
    4. Equazioni con i numeri complessi
    5. Disequazioni con i numeri complessi
    6. Esercizi misti Numeri complessi
  12. Serie numeriche
    1. Teoria Serie numeriche
    2. Esercizi Serie a termini positivi
    3. Esercizi Serie a termini di segno variabile
    4. Esercizi Serie geometriche e telescopiche
  13. Successioni di funzioni
    1. Teoria Successioni di funzioni
    2. Esercizi Successioni di funzioni
  14. Serie di funzioni
    1. Teoria Serie di funzioni
    2. Esercizi Serie di funzioni
  15. Serie di potenze
    1. Teoria Serie di potenze
    2. Esercizi Serie di potenze
  16. Serie di Fourier
    1. Teoria Serie di Fourier
    2. Esercizi Serie di Fourier
  17. Trasformata di Fourier
    1. Teoria Trasformata di Fourier
    2. Esercizi Trasformata di Fourier
  18. Funzioni di più variabili
    1. Teoria Funzioni di più variabili
    2. Massimi e minimi liberi e vincolati
    3. Limiti in due variabili
    4. Integrali doppi
    5. Integrali tripli
    6. Integrali di linea di prima specie
    7. Integrali di linea di seconda specie
    8. Forme differenziali e campi vettoriali
    9. Teorema di Gauss-Green
    10. Integrali di superficie
    11. Flusso di un campo vettoriale
    12. Teorema di Stokes
    13. Teorema della divergenza
    14. Campi solenoidali
    15. Teorema del Dini
  19. Equazioni differenziali lineari e non lineari
    1. Teoria equazioni differenziali lineari e non lineari
    2. Equazioni differenziali lineari e non lineari del primo ordine omogenee
  20. Equazioni differenziali lineari
    1. Del primo ordine non omogenee
    2. Di ordine superiore al primo,a coefficienti costanti,omogenee
    3. Di ordine superiore al primo,a coefficienti costanti,non omogenee
    4. Di Eulero,di Bernoulli,di Clairaut,di Lagrange e di Abel
    5. Non omogenee avente per omogenea associata un’equazione di Eulero
    6. Sistemi di EDO
  21. Equazioni differenziali non lineari
    1. A variabili separabiliO
    2. A secondo membro omogeneo
    3. Del tipo y’=y(ax+by+c)
    4. Del tipo y’=y(ax+by+c)/(a’x+b’y+c’)
    5. Equazioni differenziali esatte
    6. Mancanti delle variabili x e y
    7. Cenni sullo studio di un’assegnata equazione differenziale non lineare
    8. Di Riccati
    9. Cambi di variabile: simmetrie di Lie
  22. Analisi complessa
    1. Fondamenti
    2. Funzioni olomorfe
    3. Integrale di Cauchy e applicazioni
    4. Teorema della curva di Jordan e teorema fondamentale dell’Algebra
    5. Teorema di inversione di Lagrange
    6. Teorema dei Residui
    7. Funzioni meromorfe
    8. Prodotti infiniti e prodotti di Weierstrass
    9. Continuazione analitica e topologia
    10. Teoremi di rigidità di funzioni olomorfe
    11. Trasformata di Mellin
  23. Equazioni alle derivate parziali
    1. Equazioni del primo ordine
    2. Equazioni del secondo ordine lineari
    3. Equazioni non-lineari
    4. Sistemi di PDE
  24. Funzioni speciali
    1. Funzione Gamma di Eulero
    2. Funzioni Beta,Digamma,Trigamma
    3. Integrali ellittici
    4. Funzioni di Bessel
    5. Funzione zeta di Riemann e funzioni L di Dirichlet
    6. Funzione polilogaritmo
    7. Funzioni ipergeometriche
  25. Analisi funzionale
    1. Misura e integrale di Lebesgue
    2. Spazi Lp,teoremi di completezza e compattezza
    3. Spazi di Hilbert,serie e trasformata di Fourier
    4. Teoria e pratica dei polinomi ortogonali
    5. Spazi di Sobolev
  26. Complementi
    1. Curiosità e approfondimenti
    2. Compiti di analisi
    3. Esercizi avanzati analisi
  27. Funzioni Convesse

 
 

Tutti gli esercizi di geometria

In questa sezione vengono raccolti molti altri esercizi che coprono tutti gli argomenti di geometria proposti all’interno del sito con lo scopo di offrire al lettore la possibilità di approfondire e rinforzare le proprie competenze inerenti a tali argomenti.

Strutture algebriche.





 
 

Risorse didattiche aggiuntive per approfondire la matematica

Leggi...

  • Math Stack Exchange – Parte della rete Stack Exchange, questo sito è un forum di domande e risposte specificamente dedicato alla matematica. È una delle piattaforme più popolari per discutere e risolvere problemi matematici di vario livello, dall’elementare all’avanzato.
  • Art of Problem Solving (AoPS) – Questo sito è molto noto tra gli studenti di matematica di livello avanzato e i partecipanti a competizioni matematiche. Offre forum, corsi online, e risorse educative su una vasta gamma di argomenti.
  • MathOverflow – Questo sito è destinato a matematici professionisti e ricercatori. È una piattaforma per domande di ricerca avanzata in matematica. È strettamente legato a Math Stack Exchange ma è orientato a un pubblico con una formazione più avanzata.
  • PlanetMath – Una comunità collaborativa di matematici che crea e cura articoli enciclopedici e altre risorse di matematica. È simile a Wikipedia, ma focalizzata esclusivamente sulla matematica.
  • Wolfram MathWorld – Una delle risorse online più complete per la matematica. Contiene migliaia di articoli su argomenti di matematica, creati e curati da esperti. Sebbene non sia un forum, è una risorsa eccellente per la teoria matematica.
  • The Math Forum – Un sito storico che offre un’ampia gamma di risorse, inclusi forum di discussione, articoli e risorse educative. Sebbene alcune parti del sito siano state integrate con altri servizi, come NCTM, rimane una risorsa preziosa per la comunità educativa.
  • Stack Overflow (sezione matematica) – Sebbene Stack Overflow sia principalmente noto per la programmazione, ci sono anche discussioni rilevanti di matematica applicata, specialmente nel contesto della scienza dei dati, statistica, e algoritmi.
  • Reddit (r/Math) – Un subreddit popolare dove si possono trovare discussioni su una vasta gamma di argomenti matematici. È meno formale rispetto ai siti di domande e risposte come Math Stack Exchange, ma ha una comunità attiva e molte discussioni interessanti.
  • Brilliant.org – Offre corsi interattivi e problemi di matematica e scienza. È particolarmente utile per chi vuole allenare le proprie capacità di problem solving in matematica.
  • Khan Academy – Una risorsa educativa globale con lezioni video, esercizi interattivi e articoli su una vasta gamma di argomenti di matematica, dalla scuola elementare all’università.






Document









Document